AAMC Practice Test 3 - Kaplan Test Prep

21 downloads 5631 Views 1MB Size Report
aaMc TesT 3 physical sciences—Kaplan explanaTions passage i (items ... Scope : Reactions were performed to study the chemistry of lead. ¶1: Rxn 1: Pb(NO. 3. ).
MCAT

*

kaplan explanations for

AAMC Practice Test 3

*MCAT is a registered trademark of the Association of American Medical Colleges.

© 2008 Kaplan, Inc. All rights reserved. No part of this book may be reproduced in any form, by photostat, microfilm, xerography or any other means, or incorporated into any information retrieval system, electronic or mechanical, without the written permission of Kaplan, Inc. This book may not be duplicated or resold, pursuant to the terms of your Kaplan Enrollment Agreement.

Physical Sciences

physical sciences ANSWER KEY

1. A

14. C

27. A

40. D



2. D

15. C

28. C

41. A



3. A

16. B

29. C

42. B



4. B

17. D

30. D

43. D



5. B

18. A

31. A

44. C



6. C

19. B

32. C

45. B



7. A

20. D

33. C

46. A



8. C

21. A

34. A

47. A



9. A

22. C

35. C

48. B

10. A

23. C

36. A

49. D

11. D

24. A

37. C

50. B

12. C

25. A

38. B

51. A

13. B

26. D

39. C

52. B



AAMC Test 3 Explanations

AAMC Test 3 Physical Sciences—Kaplan Explanations Passage I (Items 1–5) Passage Map Passage Type: Experiment Topic: Lead Scope: Reactions were performed to study the chemistry of lead ¶1: Rxn 1: Pb(NO3)2 + Na2SO4 → Compound A (white ppt) ¶2: Rxn 2: KI + Compound A → PbI2 (yellow ppt) ¶3: Rxn 3: PbI2 + Na2CO3 → PbCO3 (white ppt) ¶4: Rxn 4: PbCO3 + HCl → gas 1. (A) The item prompts us to analyze Reaction 4, determine the reactants and products, and balance the final equation. Each of the answer choices offers us a different set of reactants and products, so we’ll start by determining one side of the equation first. Our passage map indicates that the information we need is found in paragraph 4. Here, we are told that PbCO3, when treated with HCl, gave off a small amount of gas. As such, we know that the reactants were PbCO3 and HCl—we can thus eliminate choices (B) and (D). In order to choose between the remaining answer choices, we must analyze the products formed. Both choices (A) and (C) propose a gas as a final product, so we can’t use this information to eliminate the last remaining wrong answer choice. Choice (A) shows a substitution reaction with the products PbCl2, CO2, and H2O. This is a balanced reaction that depicts plausible products with CO2 as the emitted gas. Looking at choice (C), however, we find an unbalanced reaction with products that cannot be formed during a simple chemical reaction. This is a less plausible product proposal because it implies that the carbonate anion would split into carbon and oxygen, a process that is not chemically favored. In addition, when HCl reacts, the most likely gas formed will be H2—not Cl2. Therefore, the answer choice that shows the balanced reaction of the fourth experiment is choice (A), the correct answer. (B) Distortion. According to the passage, the reactants proposed for this experiment are PbCO3 and HCl. (C) Distortion. Although the reactants match those suggested in the passage, the products depicted in this equation are unlikely to form under normal chemical conditions. In addition, this is an unbalanced reaction. (D) Distortion. According to the passage, the reactants proposed for this experiment are PbCO3 and HCl. 2. (D) This is a rather straightforward item that asks us to determine the identity of Compound A. The answer choices depict four different salts, three of which contain a lead cation. Let’s review our map to find where Compound A was formed. According to our map, in Reaction 1, Pb(NO3)2 and Na2SO4 were mixed together, leading to the formation of Compound A, a white precipitate. As such, Compound A can only be formed from the cations and anions provided—in this case, Pb2+ or Na+, and NO3– or SO42–. Because of this, we can eliminate choice (B). Also, since the reactants were soluble, whereas Compound A is a precipitate, the identity of Compound A cannot be one of the reactants—choice (A) can also be eliminated. To decide whether Compound A is NaNO3 or PbSO4,



Physical Sciences

we can either use our knowledge of salt solubility, or check the subsequent reaction in which Compound A was used as a reactant—we’ll use both approaches. Nitrate salts are always soluble, so NaNO3 must be aqueous (not a precipitate), leaving PbSO4 as the only candidate for the formed precipitate. Additionally, according to Reaction 2, when KI is mixed with Compound A, one of the products is PbI2. Since the lead in this product must have originated from Compound A, this confirms our suspicion that Compound A contains lead. The correct answer is choice (D). (A) Faulty use of detail. This is one of the reactants—Compound A is a product of the first reaction. (B) Distortion. Compound A cannot contain iodide, since it is not present in either of the reactants. (C) Faulty use of detail. This is one of the products of Reaction 1; however, NaNO3 is soluble, and therefore would not be found as the white precipitate (Compound A). 3. (A) The item tells us that Pb(OH)2 is only slightly soluble in water. Glancing at the answer choices, we notice that our task is to determine how an increase in the pH of the solution will affect the solubility of lead hydroxide. To answer this item, we must first write the equation that describes how Pb(OH)2 dissolves in water. Pb(OH)2(s) ⇋ Pb2+(aq) + 2OH–(aq) The solid Pb(OH)2 dissociates into lead cation and hydroxide anions. How will the pH affect this solubility equilibrium? A pH of 9 is basic. Therefore, increasing the pH of a solution increases the number of hydroxide ions in solution. According to Le Châtelier’s principle, when a stressor, such as a change in concentration, is applied to a reaction, the equilibrium will shift so as to eliminate this stressor (that is, counteract the change in concentration). By increasing the pH, we are increasing the concentration of hydroxide ions. Since the hydroxide ions are on the product side, we can infer from Le Châtelier’s principle that the reaction equilibrium will shift to the left, towards the solid reactant, so as to alleviate the stress on the system. We would expect, therefore, that increasing the pH of the solution up to pH 9 will increase the amount of Pb(OH)2 solid, thus decreasing its solubility—only choice (A) mirrors our prediction, and is the correct response. (B) Opposite. The increase in pH will drive the reaction towards the solid reactant, therefore decreasing its solubility. (C) Opposite. The increase in pH will drive the reaction towards the solid reactant, therefore decreasing its solubility. (D) Opposite. The increase in pH will drive the reaction towards the solid reactant, therefore decreasing its solubility. 4. (B) This item requires a good understanding of the experimental procedure. Glancing at the answer choices, we are asked to determine in which order the three anions (I–, SO42–, and CO32–) will precipitate. To establish the relative solubility of these anions, we must follow the precipitation reactions in the passage—during a given reaction, the more soluble anion is displaced from its compound (dissolving into the solution) by the relatively less soluble anion, which will form an insoluble precipitate. We can make this comparison, since the cation used in each of the precipitation/displacement reactions is Pb2+. According to Reaction 1, our initial products are NaNO3 or PbSO4. Since nitrates are soluble, the identity of the first precipitate, Compound A, is PbSO4. During the subsequent reaction, Reaction 2, PbI2 precipitates (SO42– is displaced, and is now dissolved in solution). Lastly, during the third reaction, a PbCO3 precipitate forms (I– is



AAMC Test 3 Explanations

displaced, and is now dissolved in solution). Thus, the order in which the anions precipitate in the passage is: SO42–, I–, and CO32–. In other words, because the carbonate salt is formed last, we can infer that CO32– is less soluble than I–, which in turn is less soluble than SO42–. Therefore, we predict that the order in which the anions will be precipitated by Pb2+ is CO32–, I–, and SO42. From the given answer choices, only choice (B) correctly describes this sequence, and is thus the correct response. (A) Opposite. The order in which the anions precipitate in the passage is SO42–, I–, and CO32–. In other words, because the carbonate salt is formed last, we know that CO32– is less soluble than I–, which in turn is less soluble than SO42–. The order in which the anions will be precipitated is CO32–, followed by I–, and SO42–. (C) Opposite. The order in which the anions precipitate in the passage is SO42–, I–, and CO32–. In other words, because the carbonate salt is formed last, we know that CO32– is less soluble than I–, which in turn is less soluble than SO42–. The order in which the anions will be precipitated is CO32–, followed by I–, and SO42–. (D) Opposite. The order in which the anions precipitate in the passage is SO42–, I–, and CO32–. In other words, because the carbonate salt is formed last, we know that CO32– is less soluble than I–, which in turn is less soluble than SO42–. The order in which the anions will be precipitated is CO32–, followed by I–, and SO42–. 5. (B) The item prompts us to analyze Reaction 1 so as to determine the number of moles of Na+ present in the Na2SO4 solution. A quick glance at the answer choices confirms the calculative nature of our approach. To calculate the moles of Na+, we must first determine the number of moles of Na2SO4 that were initially present. To do this, we need to review Reaction 1. Here, we are told that 15 mL of 0.3 M Na2SO4 solution is used. How many moles does this represent? We can multiply the molarity by the volume to obtain the number of moles:

nNa2SO4 = 15 mL × 0.3 mol/L

Converting into scientific notation yields:

nNa2SO4 = 15 × 10–3 L × 3 × 10–1 mol/L

nNa2SO4 = 45 × 10–4 mol

nNa2SO4 = 4.5 × 10–3 mol = 0.0045 mol

There are 0.0045 moles of Na2SO4 used in Reaction 1. However, the item asks us to determine the number of moles of Na+ in solution. For every molecule of Na2SO4, there are 2 moles of Na+; therefore, the number of moles of sodium ion is double that of sodium sulfate.

nNa+ = 0.0045 mol × 2 = 0.009 mol

We conclude that there are 0.009 moles of Na+ in solution, which matches with the correct answer, choice (B). (A) Miscalculation. This result implies that there are four sodium ions for each Na2SO4, when in fact there are only two. (C) Miscalculation. This is the number of moles of Na2SO4. (D) Miscalculation. This is an order of magnitude error.



Physical Sciences

Passage II (Items 6–11) Passage Map Passage Type: Experiment Topic: X-rays Scope: To define and prove two types of X-ray spectra ¶1: Two types of X-ray spectra: continuous and line spectra ¶2: Continuous spectrum: free e– accelerate during collisions with ions (bremsstrahlung) ¶3: Line spectrum: an electron from a heavy atom is ejected during a collision with another electron; a photon is emitted when a higher energy electron fills in the lower energy orbital. ¶4: X-ray tube Fig. 1: X-ray tube; the cathode emits electrons ¶5: Description of X-ray tube; C, LV, A and HV defined ¶6: Relationship between wavelength and intensity Fig. 2: X-ray intensity vs. wavelength; P2 has higher wavelength and intensity than P1. 6. (C) The item requires a thorough understanding of the passage and the second figure. We are prompted to analyze Figure 2, which shows the two peaks, P1 and P2. What is the source of these sudden spikes in the intensity? To find the answer, we must read the paragraph that describes Figure 2. According to our map, the figure illustrates the line spectrum discussed in paragraph 6. In this paragraph, we are told intensity, I, “is proportional to the number of x-ray photons emitted.” When is a photon emitted? According to paragraph 3, when an electron is emitted from a heavy atom, another electron that has a higher energy will fill in the empty lower energy orbital. This results in the emission of a photon. Knowing this, we can infer that the two peaks in Figure 2 correspond to a drastic increase in the number of photons emitted during the line spectrum experiment. With this prediction in mind, we select choice (C) as the correct response. (A) Faulty use of detail. Bremsstrahlung is not produced from photons—according to paragraph 6, the intensity of the spikes in Figure 2 is proportional to the number of emitted photons. (B) Opposite. According to paragraph 6, photons are emitted, not absorbed. (D) Opposite. According to paragraph 2, the electromagnetic radiation produced by the acceleration of electrons in a magnetic field is the definition of Bremsstrahlung and is not produced from photons. According to paragraph 6, the intensity of the spikes in Figure 2 is proportional to the number of emitted photons. 7. (A) We are asked to determine the amount of power consumed—a quick glance at the answer choices reveals that we must do some calculations. The item stem makes reference to Figure 1, informing us that the current in the circuit is 0.005 A, or 5 × 10–3 A, and that the voltage drop is 105 V. What formula allows us to calculate the power in an electric circuit? P = IV. Using the values given to us in the item stem, we obtain the following:



AAMC Test 3 Explanations



P = IV



P = (5 × 10–3 A)(105 V)



P = 5 × 102 W

Our calculated result matches with choice (A), the correct answer. (B) Miscalculation. This is double the amount of power consumed. (C) Miscalculation. As always, be careful with your calculations. (D) Miscalculation. As always, be careful with your calculations. 8. (C) In spite of the lengthy wording, this item is actually rather straightforward. We are given the energies of the first three orbitals and asked to calculate the energy of the X-ray photon emitted when an electron moves from the second to the first energy level. Where is this discussed in the passage? According to our map, paragraph 3 explains the details of photon emission. Here, we are told that the energy of the photon emitted when an electron moves from a higher to a lower energy orbital is equal to the difference in the energy of the two orbitals. Our task in answering this question is to simply take the difference between the first, n = 1, and the second, n = 2, energy levels.

EX-ray = En1 – En2 = 1400 × 10–17 J – 240 × 10–17 J



EX-ray = 1160 × 10–17 J



EX-ray = 1.16 × 10–14 J

We conclude, therefore, that the energy of the X-ray emitted when an electron moves from n = 2 to n = 1 is 1.16 × 10–14 J—choice (C) is the correct answer. (A) Miscalculation. This is the energy of the photon emitted when an electron moves from n = 3 to n = 2. (B) Miscalculation. This is the energy of the second, n = 2, level. (D) Miscalculation. This is the energy of the photon emitted when an electron moves from n = 3 to n = 1. 9. (A) This is a straightforward item that tests our understanding of the passage. The keyword in the item stem is “bremsstrahlung”—we are asked to determine when this phenomenon will not occur. Glancing at the answer choices, we notice that our task is to specifically select an atom or atomic particle which will not undergo bremsstrahlung. Where does the passage mention this phenomenon? According to our passage map, such information is located in paragraph 2—bremsstrahlung is “the electromagnetic radiation produced when free electrons are accelerated during collisions with ions.” Since this is the only information we have, the correct answer must not fit this criterion—it will not be an ion. Choices (B) and (C) are ions, while a proton, choice (D), is an atomic particle with charge. Choice (A), the non-ionized atom, cannot produce a bremsstrahlung, and is thus the correct answer. (B) Opposite. Bremsstrahlung will occur between electrons and ions. He2+ is an ion. (C) Opposite. Bremsstrahlung will occur between electrons and ions. Li1+ is an ion. (D) Opposite. Bremsstrahlung will occur between electrons and ions. Protons are positively-charged atomic particles.



Physical Sciences

10. (A) This item asks us to identify the way to increase the kinetic energy of the electrons colliding with the anode. The answer choices make reference to the HV and the LV components of the circuit. To understand which variable is responsible for generating the kinetic energy of the electrons, let’s quickly review the passage—our passage map indicates that these terms are defined in paragraph 5. HV is described as a power supply that regulates electron acceleration. LV, on the other hand, is a power supply that regulates the temperature of the filament. Which of these would affect the kinetic energy of the electrons? The kinetic energy of any particle is proportional to the square of its speed, while the speed is directly related to the acceleration of the particle. Knowing this, the voltage of LV will have no affect on the kinetic energy of the electrons—we can eliminate choices (C) and (D). To increase the kinetic energy, we must increase the speed of the electrons. The only way to do this is to increase the acceleration of the particles by increasing the voltage of HV—choice (A) is the correct answer. (B) Opposite. This would lower the kinetic energy of the electrons by lowering their acceleration, and, subsequently, their speed. (C) Faulty use of detail. LV is a power supply that controls the temperature of the filament. This will not affect the kinetic energy of the electrons. (D) Faulty use of detail. LV is a power supply that controls the temperature of the filament. This will not affect the kinetic energy of the electrons. 11. (D) The item prompts us to analyze Figure 2 so as to determine which of the two peaks represents the more probable event. We can categorize the answer choices into two groups: those that argue for either P1 or P2 as the more probable event, with an explanation based on wavelength or intensity of the peaks. Without any analysis, we can immediately eliminate choice (A), since it states that P1 has a longer wavelength than P2, which is clearly not true based on Figure 2. To continue, let’s quickly review what is described in Figure 2. According to paragraph 6, the intensity of the peaks is directly related to the number of X-ray photons emitted at a certain wavelength. As such, a peak with a higher intensity reflects an event in which more photons were emitted. Since a photon is emitted for every change in orbital that an electron makes, we can safely assume that more photons emitted implies that more electrons moved from a higher to a lower energy level—this translates into a higher probability of this particular event. We predict, therefore, that the P2 peak has a higher probability of occurring because it has a higher intensity than P1—choice (D) is the correct answer. (A) Opposite. P1 has a shorter wavelength than P2. (B) Faulty use of detail. This implies that P1 will have a lower probability of occurring. (C) Faulty use of detail. While P2 has a longer wavelength than P1, this does not explain why P2 has a higher probability.

Discrete Set I (Items 12–15) 12. (C) We are asked to calculate the amount of work done when pushing an object—the numerical nature of the answer choices confirms that some calculations are required. Work is equal to the product of the force, the distance over which this force acts, and the cosine of the angle between the force and the direction of movement. We can calculate the work according to the formula:



AAMC Test 3 Explanations

W = Fd cos θ We are given the value for the force, the distance over which the force is acting, and we are told that the force is pushing the object horizontally, so θ = 0, making cos θ = 1. In our case, work is simply the product of the force and distance: W = 20 N × 10 m = 200 J Our result matches with the correct answer, choice (C). (A) Miscalculation. This is the mass of the block. (B) Miscalculation. This is the amount of work done to move the block a distance of 5 m. (D) Miscalculation. This answer would be incorrectly obtained if you multiplied the mass of the block and the distance. 13. (B) This item tests our understanding of phase changes, specifically evaporation. Let’s start by reviewing the factors necessary for vaporization to occur. According to the item stem, we are told that liquids evaporate when the molecules overcome the attractive forces between them. From our general knowledge, we know that to evaporate a liquid we must increase its temperature until the molecules at the surface change phase to become gaseous. How does temperature relate to attractive forces? Temperature, by definition, is a measure of the average kinetic energy of the molecules. By increasing the temperature of a liquid, we are also increasing its kinetic energy. Since the liquid particles are moving with greater velocity, it becomes easier for them to break the intermolecular attractive forces that are holding them together in the liquid lattice—such an infusion of energy will eventually lead to the desired phase change. Thus, evaporation occurs when the molecules in the liquid attain sufficient kinetic energy so as to overcome the attractive forces holding them together—choice (B) is the correct answer. (A) Distortion. Resonance occurs when a wave has the same frequency as the fundamental frequency (during the first harmonic) of that wave. (C) Distortion. Surface tension is a property of molecules in a liquid that reflects the strength of the attractive forces within the liquid. (D) Distortion. Potential energy refers to static molecules, not those that are moving. 14. (C) We recognize this item as a kinematics question. The answer choices are expressed in seconds—our task is to calculate the time that it takes the runner to travel a certain distance. As with any kinematics question, the safest approach is to write down what variables we are given, which one we have to determine, and then decide which equation is most applicable. In this case, we are told that the runner starts from rest, so vo = 0 m/s. We are also given the acceleration, a, and the change in distance, Δx. The variable that we need to calculate is time, t. The most applicable equation is: Δx = vot + __ ​ 1  ​at2 2 Since vo = 0 m/s, we can simplify: Δx = __ ​ 1  ​at2 2



Physical Sciences

Solving for time:

____



t = ​  ____ ​ 2Δx    a ​  ________ 2 × 3 m  t = ​  ​ _________    ​  1.5 m/s2





 ​

________

 ​

t = ​ √ 2 × 2 s2   ​



t = ​ √4s2 ​ 



t=2s

____

Choice (C) is the correct answer. (A) Miscalculation. This is how long it would take the runner to travel half the distance. (B) Miscalculation. As always, be careful with your calculations. (D) Miscalculation. As always, be careful with your calculations. 15. (C) In glancing at the figure, we should immediately notice that this item is testing us on our understanding of forces and vector additions. Indeed, after reading the item stem, we confirm that our task is to determine the magnitude of force F, directed at an angle θ = arctan 0.75, that will provide translational equilibrium (no net force). To determine the magnitude of F, we have to closely analyze the figure. We notice that although the 3-N force is pointing down, the 4-N force is pointing to the left. Since θ = arctan 0.75, we can set up the following equality: tan 3N θ = ​ ___  ​ = 0.75. This confirms that the angle θ is found between the 4-N force and F, as the diagram below shows: 4 N 

F

F = 3N

θ F = 4N Based on all of the information above, we conclude that if we were to add the 3-N and the 4-N vectors together, we would obtain a resultant force that has the same magnitude and direction as F. Therefore, we should be able to calculate the magnitude of F by using the Pythagorean Theorem.

F2 = 42 + 32



F2 = 16 + 9 = 25 N



F=5N

F has a value of 5 N—the correct answer is choice (C). (A) Miscalculation. This is the magnitude of the force that points down. (B) Miscalculation. This is the magnitude of the force that points to the left.



AAMC Test 3 Explanations

(D) Miscalculation. This answer would be incorrectly obtained if the 4-N and the 3-N forces were simply summed—we must use vector analysis to add the forces.

Passage III (Items 16–21) Passage Map Passage Type: Experiment Topic: Melting and freezing points Scope: To determine the melting and freezing points of acetamide ¶1: Exp. 1 – Melting: Acetamide melts at 80°C; temperature is constant during the melting process. ¶2: Exp. 2 – Freezing – Trial 1: Melted acetamide from Exp. 1 is left to freeze in 20°C air; temperature is constant during freezing. ¶3: Exp. 2 – Freezing – Trial 2: Solid acetamide from Trial 1 was melted, then cooled in 20°C water. Fig. 1: Melting and freezing behavior of acetamide 16. (B) This item requires us to have a good understanding of the experimental procedures used in Trials 1 and 2. Specifically, we are asked to determine the difference between these two experiments—let’s review our map. According to the information in paragraphs 2 and 3, the same acetamide was used in both trials. In Trial 1, the melted acetamide was left to cool in air at 20°C. However, in the second trial, the acetamide was cooled in water at 20°C. In both trials, acetamide melted at 80°C. After analyzing these procedures, the only significant difference is found in the surroundings in which acetamide was left to cool. Indeed, when the acetamide was left to cool in air, the time for freezing was considered to be excessive, while the time for freezing in water was much quicker. This is reasonable, since the temperature of the air was variable, while that of the water was uniform—choice (B) is the correct answer. (A) Distortion. The same amount of acetamide was used in Trials 1 and 2. (C) Distortion. Both trials were started at an environmental temperature of 20°C. (D) Distortion. Both trials deal with the freezing of acetamide, not melting. 17. (D) The item prompts us to analyze the methodology of Experiment 1 to determine how a change in the ambient temperature would affect the melting of acetamide. The answer choices can be divided into two categories: those that discuss the amount of water needed and those that analyze the melting behavior of acetamide. Let’s start by reviewing the passage, specifically the first experiment. We are told that the melting point of acetamide is 80°C. In Experiment 1, a test tube containing solid acetamide is placed in a beaker with boiling water at 100°C. Since the ambient temperature (boiling water) is higher than its melting temperature, acetamide will melt. Therefore, the melting of the acetamide depends on the temperature of the water, and not the amount of water in the beaker. While the amount of water might influence how much acetamide can dissolve into aqueous solution, it will not have an effect on the melting process. Knowing this, we can eliminate choices (A) and (B). The item stem asks us to determine what would happen if the temperature of the water was at 90°C instead of 100°C. Since 90°C is still

10

Physical Sciences

above the melting point of acetamide (80°C), we still expect it to melt; however, the lower ambient temperature will cause the solid to take longer to melt—choice (D) is the correct answer. (A) Faulty use of detail. The amount of water might influence how much acetamide can dissolve into aqueous solution, but it would not have an effect on whether or not it can melt. Regardless of the amount of water, the solid acetamide can transition from solid to liquid if the temperature of the water is higher than the melting point. (B) Faulty use of detail. The amount of water might influence how much acetamide can dissolve into aqueous solution, but it would not have an effect on whether or not it can melt. Regardless of the amount of water, the solid acetamide can transition from solid to liquid if the temperature of the water is higher than the melting point. (C) Opposite. The acetamide would melt because the temperature of the water is still higher than the melting temperature of acetamide. 18. (A) This is a rather open-ended item that tests our understanding of the scientific method. We are asked to identify the aspect of the experimental procedure that allows the results of the two experiments to be useful. The first two answer choices refer to the control of the freezing and melting points, whereas the last two relate the amount of acetamide to the time and temperature of the melting and freezing processes. Although we cannot make a very specific prediction, it is worth glancing at our passage map to remind ourselves of the key similarities and differences between the two experiments. Both experiments have very well established conditions for the melting and freezing points of acetamide (the temperature of the solution is constantly being measured during melting and freezing, and the time it takes for acetamide to change phases is also closely monitored). As such, the experimental conditions are controlled; this allows us to eliminate answer choice (B). As far as the mass of acetamide that is used, the same amount of solid is used throughout the entire passage. Since the amount of acetamide did not differ between different trials and experiments, the results cannot offer any correlation between the amount of solid and the time or the temperature for freezing and melting. In other words, the amount of acetamide is irrelevant to the experimental conditions. Only choice (A) correctly explains the importance of the experiments: controlled conditions allow the results of the experiments to be meaningful. (B) Opposite. The experimental conditions were very closely monitored, as the temperature of the water, masses, volumes, and freezing and melting points were constantly measured. (C) Distortion. Since the amount of acetamide did not differ between different trials and experiments, the results cannot offer any correlation between the amount of solid and the temperature of freezing and melting. (D) Distortion. Since the amount of acetamide did not differ between different trials and experiments, the results cannot offer any correlation between the amount of solid and the time needed for freezing and melting. 19. (B) The item presents us a hypothetical situation and requires us to determine how the results of the experiment would be affected. Specifically, we are asked to predict the effect of obtaining temperature measurements less often—every minute instead of every 30 seconds. The answer choices offer different freezing times—it is our task to establish what effect such measurement changes would have on the time it takes to freeze the acetamide. Let’s start by reviewing the first trial in Experiment 2, as the question stem prompts us to. In Trial 1, the experimenters analyzed the freezing behavior of acetamide. We are told towards the end of the paragraph that the acetamide was completely solid after 23 minutes, whereas the temperature readings were performed every 30 seconds. If we change the measurement intervals, what effect will this have on the freezing time? None! How often we measure the temperature has nothing to do with how fast the acetamide freezes. The freezing behavior

11

AAMC Test 3 Explanations

is inherent to the acetamide and the surrounding temperatures, but is not affected in any way by how often we measure the temperature. We predict, therefore, that it would still take 23 minutes for the acetamide to melt, regardless of how often we measure the temperature—choice (B) is the correct answer. (A) Distortion. This is half the time it takes acetamide to melt. Even if we measure the temperature less often (every minute instead of every 30 seconds), the acetamide will still freeze with the same speed. (C) Distortion. This is twice the time it takes acetamide to melt. Even if we measure the temperature less often (every minute instead of every 30 seconds), the acetamide will still freeze with the same speed. (D) Distortion. The freezing point of acetamide is not affected by how often we make the temperature measurements. 20. (D) The item stem asks us to analyze a primary difference between the two trials in Experiment 2. Specifically, we have to determine why the acetamide was placed in hot water at the beginning of the second trial, but not during the first trial. In order to answer this, we need to review our passage map. The acetamide used in the first trial of Experiment 2, was taken from Experiment 1. In the first experiment, the acetamide was melted. So at the beginning of the first trial of Experiment 2, the acetamide was liquid. During Trial 1, the acetamide was frozen, so at the beginning of Trial 2, the acetamide was present in its solid form. Since the purpose of Trial 2 was to monitor the freezing point of acetamide, the experimenters had to first melt it. Therefore, hot water was used in Trial 2 in order to melt the acetamide so as to maintain the same experimental conditions during both trials—choice (D) must be the correct answer. (A) Distortion. Trial 1 used air, not water, to cool/freeze the acetamide (not boiling water). (B) Faulty use of detail. Although this observation is correct, it does not explain why the acetamide had to be melted for Trial 2 and not Trial 1. (C) Faulty use of detail. Although this observation is correct, it does not explain why the acetamide had to be melted for Trial 2 and not Trial 1. 21. (A) The item requires us to analyze Figure 1 in order to determine how a plot of Trial 1 would compare to that of Trial 2 (data which is already presented in the graph). The answer choices discuss the differences between the two trials in terms of the slopes of the graph and the time axis of the graph. Let’s look at Figure 1. On the vertical axis, the temperature of acetamide is plotted, while on the horizontal axis, the timing (in seconds), which accompanies the melting/freezing, is recorded as well. Two lines are already graphed in the figure—we want to focus on Trial 2 from Experiment 2. This is the slope indicated by square data points—since Experiment 2 measures freezing time, the measured temperature should decrease, thereby having a negative slope. How did Trial 1 differ from Trial 2? According to paragraph 3, “the freezing time (of Trial 1) was considered excessive, so another trial was completed.” Indeed, the acetamide in Trial 1 froze after 23 minutes. How about the timing for Trial 2? According to Figure 1, the acetamide in Trial 2 had finished freezing after 210 seconds, or 3 minutes. We know this because the temperature begins to descend at this time in the graph, an indication that the phase change is complete. We’ve ascertained how the timing would be different, but what about the slope? Since we are measuring the rate at which acetamide freezes in Trial 1, we expect the slope to be negative, just like that of Trial 2. With this prediction in mind, we choose choice (A) as the correct answer.

12

Physical Sciences

(B) Opposite. According to the passage, it takes 23 minutes for acetamide to freeze. The time axis on the graph is expressed in seconds and runs up to 270 seconds. Considering that there are 60 seconds in 1 minute, 270 seconds is about 4–5 minutes. Therefore, the data for Trial 1 would not fit on the given figure. (C) Opposite. Both Trial 1 and Trial 2 analyze the freezing behavior of acetamide. This implies that the temperature of acetamide constantly decreases throughout the trials, so the slope has to be negative. (D) Opposite. Both Trial 1 and Trial 2 analyze the freezing behavior of acetamide. This implies that the temperature of acetamide constantly decreases throughout the trials, so the slope has to be negative.

Passage IV (Items 22–27) Passage Map Passage Type: Information Topic: Harmonics Scope: Characteristics of harmonics ¶1: Timbre depends on number and strength of harmonics. Fig. 1: Harmonics of a complex tone; 1a – the first three harmonics; 2a – summation of the first two harmonics; 3a – summation of the first three harmonics ¶2: Figure 2a and 2b – two adjacent tones from a bassoon Fig. 2: Pressure and amplitude of harmonics of two different bassoon tones 22. (C) This is an item that requires us to analyze Figure 1 so as to determine which of the three waves has the shortest period. A glance at the answer choices reveals that we must choose the harmonic that corresponds to the shortest period. The period of a wave is the time it takes for the wave to complete a full waveform; the period is also the inverse of the frequency. Although the item asks us to determine the wave with the shortest period, it might be faster to think about the harmonic that has the highest frequency. How is the frequency of a wave related to the harmonic number? By definition, a higher harmonic describes a higher frequency. In other words, between the three harmonics, the third harmonic has to have the highest frequency and therefore the lowest period (inverse relationship). Let’s verify this answer by analyzing Figure 1a, where all three harmonics are depicted. Out of the three waves, the one that has the short dashes, the third harmonic, has the lowest period. We have confidently proven choice (C) as the correct answer. (A) Opposite. This will have the longest period. (B) Opposite. This will have a period shorter than the first harmonic, but longer than the third harmonic. (D) Opposite. The period of this complex wave (summation of the three harmonics) is the same as the period of the first harmonic found in 1a. Therefore, it will have the longest period. 23. (C) This item asks us to analyze Figure 1a to interpret what the intersection of all three harmonics represents. The answer choices refer to the wave phase and the displacement away from equilibrium. Let’s take a look at the figure. At the second point where all the waves intersect, we notice that they are all situated exactly on the time

13

AAMC Test 3 Explanations

axis, that is, right at the point of zero displacement. Without any other description, we can select answer choice (C) as the correct response. (A) Distortion. This intersection point represents the point of zero displacement. When determining whether two or more waves are in phase, we must determine how their crests and troughs superimpose—at this intersection point, there are no crests or troughs. (B) Distortion. This intersection point represents the point of zero displacement. When determining whether two or more waves are in phase, we must determine how their crests and troughs superimpose—at this point, there are no crests or troughs. (D) Distortion. The maximum displacement is also known as the amplitude of the wave. Clearly, the wave form rises higher than the second point of their intersection, where the harmonic waves are at zero displacement. 24. (A) The item stem provides us with the frequency of the first harmonic in Figure 2 and requires us to determine the period of the second harmonic. The answer choices offer different numerical values for the period of the wave. Since this question has two parts, let’s break it down into smaller pieces. Our first task is to determine how the second harmonic is related to the first harmonic. Secondly, we must find out how to calculate the period of the second wave using the frequency of the first. Each successive harmonic will have a frequency that will be equal to the frequency of the fundamental harmonic (the first harmonic) multiplied by the number of the harmonic. If the frequency of the first harmonic is 100 Hz, then the frequency of the second harmonic must be 2 × 100 Hz = 200 Hz. What does this imply about the period of the second harmonic? The period of a wave is the inverse of its frequency. We can calculate the period of the second harmonic as such:

f = 200 Hz T = __ ​ 1 ​  f ____ T = ​  1   ​  200 T = 0.005 seconds

The only answer choice that matches our prediction is choice (A). (B) Miscalculation. This is the period of the first harmonic. (C) Miscalculation. This is an order of magnitude error. (D) Miscalculation. This is the numerical value for the frequency of the second harmonic. 25. (A) In spite of its apparent complexity, this item is rather straightforward. We are asked to analyze Figure 1 to determine the relative amplitudes of the three harmonics. Indeed, the answer choices offer us four possible diagrams that compare the amplitudes among the different harmonic waves. How can we use the information in Figure 1 so as to determine the relative amplitudes? The amplitude of a wave is simply the maximum displacement from its equilibrium position. Therefore, the relative amplitudes can be determined by comparing the height of the crests (or the depth of the troughs). Looking at Figure 1a, we notice that the longest wave, that of the first (fundamental) harmonic, has the highest crest, while the second and third harmonics share a smaller, yet identical amplitude. With this in mind, choice (A) must be the correct answer, as it is the only one which matches our predictive observations. (B) Distortion. The first harmonic has the greatest amplitude.

14

Physical Sciences

(C) Distortion. When compared to the first harmonic, the second and third harmonics share a smaller, yet identical amplitude. (D) Distortion. When compared to the first harmonic, the second and third harmonics share a smaller, yet identical amplitude. 26. (D) This item asks us to add a fourth harmonic to Figure 1 and determine how it will compare to the other three waves. The answer choices are qualitative comparisons that argue either for a difference in amplitude or in frequency. Let’s look at amplitude first. The amplitude of a wave is simply the maximum displacement from its equilibrium position. Looking at Figure 1a, we notice that the longest wave, that of the first (fundamental) harmonic, has the highest amplitude, whereas the second and third harmonics share a smaller, yet identical amplitude. Because the second and third harmonics have identical amplitudes, we do not know if the amplitude of the fourth harmonic would be lower or higher—we can eliminate choices (A) and (B). With regard to the frequency, we know that as the harmonic number increases, the frequency of the wave also increases (the period decreases). Therefore, the fourth harmonic would have a higher frequency than either one of the first three harmonics—the correct answer must be choice (D). (A) Distortion. Because the second and third harmonics have identical amplitudes, we do not know if the amplitude of the fourth harmonic would be lower or higher. (B) Distortion. Because the second and third harmonics have identical amplitudes, we do not know if the amplitude of the fourth harmonic would be lower or higher. (C) Opposite. A higher harmonic implies a higher frequency of the wave. 27. (A) Glancing at the answer choices, we notice that this item asks us to decide which harmonic period is similar to that of the waveform depicted in Figure 1c. The period of a wave is the time it takes for one waveform (one wavelength) to pass. As such, the waveform in Figure 1c requires the same amount of time to complete its period (or wavelength) as the wave form in Figure 1b, and the first (fundamental) harmonic in Figure 1a. We know that the solid line in Figure 1a is indicative of the first harmonic since the first harmonic corresponds to the lowest frequency/highest period (due to the inverse nature of their relationship). As such, choice (A) is the correct answer. (B) Opposite. The period of this wave is intermediate between the first and third harmonics. (C) Opposite. The period of this wave is much too short when compared to the waveform found in Figure 1c. (D) Distortion. The period of a wave cannot be calculated simply by adding the periods of the different waves that it is made of.

15

AAMC Test 3 Explanations

Passage V (Items 28–32) Passage Map Passage Type: Experiment Topic: The photoelectric effect Scope: Using the photoelectric effect to determine the conversion efficiency of two materials ¶1: The photoelectric effect described, equation given; conversion efficiency defined ¶2: Experiment to measure the conversion efficiency of Materials A and B Table 1: Results for Material A and B; wavelength and conversion efficiency 28. (C) This item requires us to analyze Table 1 so as to determine the frequency at which the conversion efficiency of Material B is 0.42. All of the answer choices are expressed in Hertz, and they differ by a few orders of magnitude. Let’s go back to Table 1 and find the data point that we are interested in. The second to last row shows the experimental results for Material B in which the conversion efficiency is 0.42. Thus, using the conversion efficiency, we now know that the wavelength at which this value was obtained. How can we calculate the frequency? What equation do we typically use to relate frequency and wavelength? We know that the product of the wavelength and the frequency is equal to the speed of light in a vacuum. Thus, we can calculate the frequency according to the following equation:

λf = c f = __ ​ c  ​ λ

The speed of light in a vacuum is 3 × 108 m/s, and the wavelength is given in the table as 1.06 × 10–6 m. Since the answer choices differ by several orders of magnitude, we can approximate the wavelength to 1 × 10–6 m. The frequency can be calculated as: 8



3 × 10 m/s f = ​ ___________      –6

 ​

f = 3 × 1014 Hz

1 × 10

m

The answer choice that most closely resembles our prediction is choice (C), the correct answer. (A) Miscalculation. As always, be careful with your calculations. (B) Miscalculation. This answer would be incorrectly obtained if we multiplied the speed of light and the wavelength. (D) Miscalculation. As always, be careful with your calculations. 29. (C) Our task in answering this item is to determine the purpose of the absorption coating. The answer choices offer four different possible reasons, but only one of them is supported by the passage. Where did we discuss the absorption coating? A quick passage scan reveals its mention in the second paragraph. Here, we are told that the “photoelectric material had a coating that maximized the absorption of light at certain frequencies.” How will this affect the results? The physicist wanted to measure the conversion efficiency of two materials. The

16

Physical Sciences

conversion efficiency, according to the first paragraph, is the “fraction of light incident on the surface of a material that is converted into electrical energy.” In other words, the physicist was interested in having as much light be absorbed by the material so as to increase the conversion efficiency of the material. We can predict, therefore, that the absorption coat served the purpose of increasing the absorption of light so as to increase the conversion efficiency. Glancing at the answer choices, we notice that only choice (C) mirrors our prediction and is thus the correct answer. (A) Distortion. There is no indication in the passage about the device being too warm. (B) Distortion. The passage does not discuss the temperature of the device and whether or not it needs to be changed. (D) Distortion. There is no mention in the passage about the magnitude of the current. 30. (D) In spite of the length of the item stem, we are simply asked to determine how a parallel set up will influence the voltage of a circuit. The answer choices give four different estimates of how the voltage in the circuit will change when two devices are connected in parallel, as opposed to having only one device. What do we know about voltage sources connected in parallel? The voltage drop across each device is the same! This means that when these two devices are connected in parallel, the voltage output will be the same as if there was only one device. Only if the two devices were connected in series would the total voltage output be the summation of each device’s contribution. The answer choice that illustrates this result is choice (D), the correct response. (A) Distortion. When two devices are connected in parallel, the voltage output will be the same as if there was only one device. (B) Distortion. When two devices are connected in parallel, the voltage output will be the same as if there was only one device. (C) Distortion. When two devices are connected in parallel, the voltage output will be the same as if there was only one device. 31. (A) The item asks us to determine the energy of an incident photon that is required to eject an electron from a material with a work function ϕ. In addition, the item stem provides us with the general formula for the energy of a photon—hf. Glancing at the answer choices, we understand that our task is to determine how this energy will compare to the work function, ϕ, and the kinetic energy of the electron, K. What information do we have in the passage that would help us answer this question? According to our map, the first paragraph discusses the photoelectric effect. When a photon of sufficient energy strikes a material, some of this energy is used to detach the electron from the material (this is the work function of the material), whereas the rest of the energy is converted into the kinetic energy of the electron. We would expect, therefore, that if we want the electron to be ejected from the material, the energy of the photon has to be bigger than the work function of the material. Analyzing the equation given to us in the first paragraph, K = hf – ϕ, we confirm that in order for the electron to be ejected, K has to be larger than zero, which further implies that hf must be bigger than ϕ. The only answer choice that matches our line of reasoning is choice (A). (B) Opposite. If hf is less than ϕ, then K will have a negative value. However, since K is the kinetic energy of the electron, which is equal to ​ _12 ​mv2, its value can only be positive. That said, if hf is less than ϕ, the electron will not be ejected.

17

AAMC Test 3 Explanations

(C) Distortion. Since hf is the energy of the photon, it will always be less than the kinetic energy of the electron because some of the energy of the photon is used to detach the electron from the material (the work function). (D) Distortion. If hf were equal to K, then ϕ would be zero—this cannot be true since every material has a work function (energy spent in the process of electron liberation). 32. (C) We are presented with a hypothetical situation and asked to determine the results of this new information. In the item stem, we are told that a new coating is applied to material A so that the conversion efficiency is independent of wavelength. We are then asked to determine which of the four given graphs will best illustrate the relationship between the conversion efficiency and the wavelength. If the conversion efficiency is independent of wavelength, we would expect the conversion efficiency to be the same value regardless of how the wavelength varies. Since the conversion efficiency is located on the vertical axis, we predict that such a graph would yield a straight, horizontal line. Indeed, such a graph shows that, regardless of the magnitude of the wavelength, the conversion efficiency remains unchanged—choice (C) is the correct answer. (A) Distortion. This graph implies that the conversion efficiency depends on the value of the wavelength, which contradicts the description in the item stem. (B) Distortion. This graph implies that there is a linear relationship between the conversion efficiency and the wavelength, which contradicts the description in the item stem. (D) Distortion. This graph implies that there is an inverse relationship between the conversion efficiency and the wavelength, which contradicts the description in the item stem.

Passage VI (Items 33–37) Passage Map Passage Type: Experiment Topic: Titrations Scope: To identify an acid by titrating it with a base ¶1: Student must identify acid that is liquid at room temperature. Table 1: Possible acids, their structure and physical properties ¶2: The acid was diluted, then titrated against NaOH. Fig. 1: The titration curve: pH vs. volume of NaOH added ¶3: Results: acid has one carboxylic acid group and MW = 85–92 g/mol 33. (C) The item prompts us to analyze the compounds in Table 1 so as to determine which two compounds would best illustrate the effect of molecular weight on melting point. The answer choices simply offer us various combinations of the compounds found in Table 1. How can we go about determining this relationship? We need to focus on comparing compounds that are most similar, especially in their functional groups. This way, the change in melting point can be attributed directly, and exclusively, to the change in molecular weight (and not the varying functional groups). From the four compounds in Table 1, propionic and butyric acid have very similar chemical

18

Physical Sciences

structures—each is an unbranched alkane chain containing a carboxylic acid. Conversely, crotonic acid has a double bond, while oxalic acid has two carboxylic acid groups. The only difference between propionic and butyric acid is their molecular weight—how does this affect melting point? The difference between the melting points of these respective compounds is roughly 14°C—this change is only due to the difference in molecular weight. Following this rationale, choice (C) must be the correct answer. (A) Distortion. These two compounds differ significantly in their chemical structure. With this extra variable, it is unclear how the difference in molecular weight alone affects the difference in the melting points of these two acids, since some of the difference can be attributed to chemical structure. (B) Distortion. These two compounds differ significantly in their chemical structure. With this extra variable, it is unclear how the difference in molecular weight alone affects the difference in the melting points of these two acids, since some of the difference can be attributed to chemical structure. (D) Distortion. These two compounds differ significantly in their chemical structure. With this extra variable, it is unclear how the difference in molecular weight alone affects the difference in the melting points of these two acids, since some of the difference can be attributed to chemical structure. 34. (A) This item asks us to calculate the hydronium ion concentration in the acidic solution before the titration with NaOH—the answer choices provide varying molarity values. Where in the passage can we find this information? Since we do not know the exact identity of the acid, we can only use the experimental results of the titration to determine the starting concentration of H3O+. In Figure 1, we notice that the graph of the titration starts at a pH of 3. At this point, the titration with NaOH has not begun (the volume of base on the x-axis is zero). As such, the solution at this point contains only acid. How does this help us determine the hydronium concentration? Since we know that the pH is 3, we can use the pH equation to calculate the H3O+ concentration:

pH = –log [H3O+]



[H3O+] = 10–pH



[H3O+] = 10–3



[H3O+] = 0.001 M

Before titration, the hydronium concentration of the unknown acid was 0.001 M; choice (A) is the correct answer. (B) Miscalculation. This is an order of magnitude error. (C) Miscalculation. As always, be careful with your calculations. (D) Miscalculation. As always, be careful with your calculations. 35. (C) This item tests us on our understanding of the freezing point of a solution. We are told that crotonic acid and oxalic acid are each dissolved in water and adjusted to the same pH with NaOH. Our task to determine which of the two acids will have a lower freezing point. Each of the answer choices argues for either one of the two compounds and presents supporting evidence for the result. Let’s start by determining what factors influence the freezing point of a solution. Freezing point is a colligative property, meaning that the properties of the solution will not depend so much on the identity of the particles (molecular weight, etc.), but rather on the number of particles in solution. The more particles that exist in a solution, the lower the freezing point of that solution. The most

19

AAMC Test 3 Explanations

common example of freezing point depression is the significant decrease in the freezing point of water that results when salt is added to it; this is why roads are sprinkled with salt in the winter, so that the water that melts from the snow does not freeze until an even colder temperature is reached. Knowing this, it is now our task to decide which acid solution has the lowest freezing point by establishing which solution contains the highest number of particles. Each solution contains the same amount of acid (0.1 moles) and is at the same pH. However, because of the identities of the acid, there is a discrepancy in how much NaOH is added. According to Table 1, crotonic acid is monoprotic, and will thus require one equivalence of NaOH for each mole of acid. Oxalic acid, on the other hand, is diprotic, since it contains two carboxylic acid groups. Because of this, oxalic acid will require twice as much NaOH than crotonic acid to obtain the same pH. In light of this, we recognize that oxalic acid has many more solute particles dissolved in solution than crotonic acid. Thus, we expect oxalic acid to have a lower freezing point—choice (C) is the correct answer. (A) Opposite. Crotonic acid is monoprotic and thus contains fewer particles in solution (less NaOH is used to bring the solution to a pH of 4.7). Therefore, it would have a relatively higher freezing point. (B) Opposite. Crotonic acid is monoprotic and thus contains fewer particles in solution (less NaOH is used to bring the solution to a pH of 4.7). Therefore, it would have a relatively higher freezing point. (D) Distortion. The oxalic acid is diprotic and thus requires two equivalents of NaOH per carboxylic acid group. This indicates that oxalic acid has more solute particles in solution than crotonic acid. 36. (A) The item stem asks us to refer back to Figure 1 so as to determine the pH when the R–COOH and R–COO– species are present in equal concentrations. Although the answer choices give us quite specific values for the pH at which this occurs, the numbers are distant enough from each other that we only need an approximation to obtain the correct answer. Let’s analyze the item stem. The point where both the acid and its conjugate base, R–COOH and R–COO–, are present is known as the buffer region. In the middle of this buffer region, the concentrations of the two species will be exactly equal. Moving down the curve towards the more acidic pH, the R–COOH species predominates, whereas when moving up along the curve towards a more basic pH, the R–COO– is the predominant species. The buffer region of a titration is the region along the graph where the pH stays relatively constant, despite the addition of the titrant (NaOH). The combination of an acid and its conjugate base allows this buffer region to resist changes in pH by neutralizing any base or acid added. As such, the buffer region is indicated as a flat, horizontal portion on the diagram. Indeed, Figure 1 depicts a relatively flat portion of the graph at a pH range of 4–5. At some point in this range, the concentrations of R–COOH and R–COO– will be equal—only choice (A) fits with this prediction, and is therefore the correct answer. (B) Distortion. This pH is located in between the buffer region and the equivalence point—where the R–COO– species is dominant. (C) Distortion. This is the equivalence point, where equal amounts of acid and base are present. (D) Distortion. This is past the equivalence point, where the base starts to predominate in solution (there is more NaOH than acid). 37. (C) The item asks us to determine why crotonic acid cannot be considered as a possible candidate for the unknown acid. Each of the answer choices attempts to offer an explanation, but only one of them gives a valid reason. Let’s review what characteristics are known regarding the unknown acid. According to our passage map, paragraph 3 informs us that the acid has only one –COOH group and a molecular weight of 85–92. With this info, crotonic

20

Physical Sciences

acid is still a candidate for the identity of the unknown. Where else in the passage can we find additional clues about the identity of the acid? According to our map, paragraph 1 describes the unknown acid as being “liquid at room temperature.” Is crotonic acid a liquid at room temperature? Since room temperature is considered to be 25°C, in order for a compound to be liquid at room temperature, its melting point/freezing point has to be lower than 25°C. According to Table 1, crotonic acid’s melting point is 71.6°C. As such, at room temperature, crotonic acid is found as a solid—crotonic acid cannot be the identity of the unknown acid; only choice (C) provides the correct rationale. (A) Distortion. There is nothing in the passage that indicates the strength of the acids. (B) Distortion. Since it contains a –COOH group, crotonic acid could easily form hydrogen bonds with water, allowing it to dissolve in an aqueous solution. (D) Opposite. Crotonic acid does have a molecular weight of 86.09 g/mol. However, the last paragraph states that the acid must have a molecular weight of 85–92 g/mol, so a molecular weight of 86 g/mol would suggest that crotonic acid could be the identity of the unknown compound.

Discrete Set II (Items 38–41) 38. (B) Upon first glance, this item seems to test radioactive decay. However, after looking at the answer choices, we notice that our task is to calculate the speed of a particle after a spontaneous decay process. We are told that radium decays into radon, an alpha particle and a gamma particle. An alpha particle is a helium atom, or a particle with 2 protons and a mass number of 4, whereas a gamma particle is a quantum of energy (a photon). The item gives us the speed of the alpha particle and asks us to calculate the speed of the radon particle, telling us to ignore the effects of the gamma particle. What concept can we use to determine the speed of a particle after a collision-like event? We can use the conservation of momentum, which utilizes the following equation:

pinitial = pfinal mradon × vradon = mα particle × vα particle mα particle × vα particle vradon = ___________________ ​   ​       m radon

While the item stem does not provide us with the masses of each of the particles, we can use the periodic table to determine the mass number of radon (MW = 222 g/mol) and helium (MW = 4 g/mol). Substituting these values into the equation, we obtain the speed of the radon particle to be:

mα particle × vα particle vradon = ___________________ ​   ​       m



4 g/mol × 1.5 × 107 m/s vradon = ​ _____________________          ​ 222 g/mol

radon

To simplify the calculation, we will round 222 g/mol to 2 × 102 g/mol:

7 6 × 10   vradon = ​ ____________    ​ 2 2 × 10 m/s



vradon = 3 × 105 m/s

21

AAMC Test 3 Explanations

Since we rounded our denominator value to a smaller number, the correct answer should be a little less than 3 × 105 m/s. The answer choice that most closely matches our prediction is choice (B), the correct answer. (A) Miscalculation. As always, be careful with your calculations when using approximations. This answer is close to the actual answer. (C) Miscalculation. As always, be careful with your calculations when using approximations. This answer is close to the actual answer. (D) Faulty use of detail. This is the speed of the helium particle. Since the radon particle is 50 times heavier, it will travel much slower (conservation of momentum). 39. (C) Glancing at the answer choices, we confirm that this item tests our understanding of reaction orders. We are given a reaction and the rate law associated with it and asked to determine the overall reaction order. How do we determine the reaction order? The reaction order is indicated by the rate law, which can only be determined experimentally. How, then, does the provided balanced equation affect the rate law? In reality, the balanced equation doesn’t provide any help in establishing the kinetics of the reaction—such information, when included, is unnecessary detail. Thus, we can calculate the overall reaction order simply by adding up the coefficients of each one of the reactants in the rate law. For example, if the coefficients add up to 3, then the reaction is third order. In our example, the rate depends on the concentrations of NO2 and F2—the coefficient of each of the exponents is 1. Adding up the coefficients of the concentrations of reactants, we obtain an overall reaction order of 2—the reaction is second order, making choice (C) the correct answer. (A) Distortion. This would be true if the rate law was R = k; independent of the reactant concentrations. (B) Distortion. This would be true if the sum of the coefficients equaled 1. (D) Distortion. This would be true if the sum of the coefficients equaled 3. 40. (D) This item tests our knowledge of the enthalpy of reactions. We are given a chemical reaction, as well as the enthalpy of formation of the lone reactant, and asked to determine the enthalpy of the overall reaction. Let’s start by categorizing the answer choices: two of them are positive (indicative of an endothermic reaction), whereas two of them are negative (indicative of an exothermic reaction). What does the provided value of heat of formation for HCl indicate? The enthalpy of formation of a compound is the energy released or absorbed when the compound is formed from its elemental compounds. Since ΔHf° for HCl is negative, it represents the energy released when H2 reacts with Cl2 to form exactly one mole of HCl. __ ​ 1  ​H2 + __ ​ 1  ​Cl2 → HCl 2 2

ΔHf° = –92.5 kJ/mol

Knowing this, the reaction given to us in the item stem represents the reverse reaction, that is, HCl decomposing into H2 and Cl2. Thus, we can conclude that the enthalpy of the given reaction should have an opposite sign to that of the enthalpy of formation of HCl—it should be positive. Based on this, we can eliminate choices (A) and (B). We now have to decide on the exact magnitude of the enthalpy of reaction. Analyzing the reaction given to us in the question stem, we notice that during this reaction, 2 moles of HCl decompose to form one mole of H2 and one mole of Cl2. Since ΔHf° is expressed in kJ/mol, when two moles of HCl decompose, the enthalpy should be twice as large. In calculating the product, the change in enthalpy will be: 2 mol × 92.5 kJ/mol = +185 kJ

22

Physical Sciences

The correct answer is choice (D). (A) Distortion. The enthalpy of reaction should be positive. (B) Distortion. The enthalpy of reaction should be positive. (C) Distortion. This value represents the enthalpy of the reaction if there was only one mole of HCl. Since we have 2 moles of reactant, we must multiply the enthalpy value by 2. 41. (A) Glancing at the answer choices, the numerical values confirm that our task is to determine the ratio of the height of the image to that of the object: ​ _oi  ​. The item stem presents us with a hypothetical situation in which an object is placed at a distance 4f from a convex (i.e., converging) lens, whereas the resulting image is located at a distance of ​ _43 ​f from the opposing side of the lens. When working with lenses and mirrors, it is important to note that the distances of the object and the image have the same values as the height of the object and image respectively. In other words, the height of the object in this question is 4, whereas the height of the image is _​ 43 ​. _ ​ 4 ​ The ratio of the heights is thus ___ ​  3  ​ = _​ 13 ​, which makes choice (A) the correct answer. 4 (B) Miscalculation. This is the inverse of the distance of the image from the opposing side of the lens, or the height of the image. (C) Miscalculation. This is the distance of the image from the opposing side of the lens, or the height of the image. (D) Miscalculation. This is the ratio of the height of the object to the height of the image.

Passage VII (Items 42–47) Passage Map Passage Type: Information Topic: Sulfuric acid Scope: Preparation properties of sulfuric acid ¶1: Preparation of Sulfuric Acid: S → SO2 → SO3 → H2SO4 Rxn 1: Sulfur trioxide reacts with water to form 98% sulfuric acid. ¶2: Properties of H2SO4: concentration, density, boiling point ¶3: Preparation of HCl and HNO3 using H2SO4 ¶4: Formation of SO2 42. (B) This item tests our understanding of oxidation numbers. Glancing at the answer choices, we confirm that our task is to determine the change in oxidation state of the sulfur atom when sulfuric acid reacts with copper. Without knowing the full reaction, we can first determine the oxidation state of sulfur in sulfuric acid, eliminating the answer choices that suggest the wrong oxidation number. Let’s now look at H2SO4. As a general rule, hydrogen has an oxidation number of +1 because it can donate an electron, while oxygen has an oxidation number of –2 because it can gain two electrons. Since the overall molecule is neutral, the overall charge of sulfuric acid must

23

AAMC Test 3 Explanations

sum to zero. As such, we can calculate the oxidation number of sulfur by adding the oxidation number of the other atoms together:

2 × OxH + OxS + 4 × OxO = 0



2 × (+1) + OxS + 4 × (–2) = 0



2 + OxS – 8 = 0



OxS – 6 = 0



OxS = +6

Since the sulfur in the sulfuric acid molecule has an oxidation number of +6, we can eliminate choices (A) and (D). In order to finish this question, we must determine the product of the reaction of sulfuric acid and copper. A quick passage scan indicates that in paragraph 4, we are told that Cu+ and SO2 are the major products of such a reaction. We’ll use the same method as above to deduce the oxidation number of sulfur in this compound, taking into consideration that there are two atoms of oxygen, each with an oxidation number of –2.

OxS + 2 × OxO = 0



OxS + 2 × (–2) = 0



OxS – 4 = 0



OxS = +4

According to this calculation, sulfur gets reduced, its oxidation number goes from +6 to +4; choice (B) is the correct answer. (A) Opposite. This choice has the oxidation states reversed. (C) Distortion. The oxidation number for sulfur in SO2 is +4, not +8. (D) Distortion. Sulfur goes from an oxidation number of +6 to +4. 43. (D) This item tests our understanding of a fundamental chemical reaction—combustion. The answer choices describe four different reactions that involve sulfur, oxygen, and hydrogen. We are asked to determine the reaction that illustrates the combustion of elemental sulfur. Combustion is a process through which a compound is burned and thus reacts with oxygen (O2)—we can eliminate choice (C). Elemental sulfur refers to the sulfur atom, S, which is not bound to any other atoms. Knowing these few points, only choice (D) can be the correct answer. (A) Distortion. Although this is a combustion reaction, H2S is not elemental sulfur. (B) Distortion. Although this is a combustion reaction, H2S is not elemental sulfur. (C) Distortion. Although this is a balanced reaction, it is not a combustion reaction—it shows the decomposition of SO3. 44. (C) This item requires us to determine the best method of increasing the yield of a reaction. Although the item stem prompts us to analyze the second step in the synthesis of sulfuric acid, it is not necessary to go back to the passage, since we are provided with enough information to determine the correct answer.The answer choices describe different attempts to increase the yield of SO3, but only one of them will be effective. What chemistry concept is being tested here? This is a typical application of Le Châtelier’s principle, which states that when a stressor (temperature, pressure, or concentration changes) is applied to a reaction, the equilibrium of the reaction

24

Physical Sciences

will shift so as to eliminate the stressor. Since SO3 is one of the products, we have to determine the most efficient way of shifting the equilibrium of the reaction towards the product side. Generally, the simplest way is to either increase the concentration of reactants or decrease the concentration of products. This prediction is sufficient to realize that choice (C), removing SO3 from the concentration mixture, is the correct answer. (A) Distortion. Since we do not have any information on whether the reaction is exothermic or endothermic, we cannot predict the effect of a temperature change. (B) Opposite. Reducing the pressure will shift the reaction towards the side with fewer moles of gas, that is, away from SO3. (D) Opposite. Since oxygen is a reactant, if removed, the equilibrium will favor the formation of reactants, therefore decreasing the amount of SO3 produced. 45. (B) The first thing to notice about this item is that the answer choices are in a fractional form—our task is to simply determine an expression for the result, not an exact number. The item asks us to determine the number of moles of water in 1 mL of H2SO4. The first step is to establish a general expression for calculating the number of moles of a compound—we can use the following equation: nH2O = ____ ​  m  ​  MW where n is the number of moles, m is the mass, and MW is the molecular weight. The molecular weight of water is 18 g/mol. According to the above equation, we expect the molecular weight of water to be located in the denominator; based on this, we can eliminate choice (D). Next, we need to calculate the mass of water in 1 mL of H2SO4 using the information from the passage. According to our map, the second paragraph discusses density and concentration of H2SO4. Here, we are told that concentrated H2SO4 is 98% H2SO4 and 2% H2O by mass. We also know that the density of H2SO4 is 1.84 g/mL. How can we use this information to determine the mass of water present? Using the formula for density, we can determine that the mass of water is: ρ = __ ​ m ​  V m = ρV where ρ is the density, m is the mass, and V is the volume. Knowing this, the mass of water should be 1.84 g/ mL × 1 mL. However, we must keep in mind that only 2% of the solution is water, so the true mass of water is only a percentage of the total mass of the solution: 1.84 g/mL × 1 mL × 0.02. The mass of water is therefore 1.84 g × 0.02. By substituting these numbers into the final expression, we get: nH2O = ____ ​  m  ​  MW ×  ​ 0.02 ____________ nH2O = ​ 1.84     18 This expression is identical to that in choice (B), the correct answer. (A) Distortion. The first fraction represents the number of moles of sulfuric acid, whereas the second shows the number of moles of water—the item stem only asks for the moles of water. (C) Distortion. According to the passage, the solution is 2% water, not 98%. (D) Distortion. According to the passage, the solution is 2% water, not 98%.

25

AAMC Test 3 Explanations

46. (A) The item presents us with a hypothetical reaction in which hydrogen is a product of the reaction between iron and sulfuric acid. Glancing at the answer choices, it is our task to determine which species is the reducing agent. A reducing agent is a compound that gets oxidized during a reaction, and as it does this, it reduces the other compounds around it. Redox reactions describe the flow of electrons that occurs between the elements that get oxidized (lose electrons) and those that get reduced (gain electrons). Based on this definition, it should be clear that the reducing and oxidizing agents should be one of the reactants. The reaction occurs as follows: Fe + H2SO4 → H2 + FeSO4 Sulfuric acid is a strong acid, as such, it dissociates in solution to yield: H2SO4 → 2H+ + SO42– Combining these equations, we get: Fe + 2H+ + SO42– → H2 + FeSO4 Since our task is to determine which compound is the reducing agent, we should establish which reactant gets oxidized—it will be the reducing agent. Which reactant loses electrons? Iron starts out with an oxidation state of zero and loses 2 electrons to reach an oxidation state of +2. Since iron is oxidized, it must be the reducing agent—choice (A) is the correct answer. Note, in general, metals tend to be reducing agents because they are more likely to lose electrons not gain them. (B) Distortion. As a product of the reaction, this cannot be a reducing agent (it must always be one of the reactants). (C) Distortion. The sulfate does not change oxidation states. (D) Distortion. In the formation of hydrogen gas, H+ (H3O+) is reduced—it is the oxidizing agent. 47. (A) This item asks us to determine which of the given four species is present in the smallest amount when 98% H2SO4 dissociates. Let’s first understand the properties of the dissociation of sulfuric acid. H2SO4 → HSO4– + H+ → SO42– + 2H+ Sulfuric acid is a diprotic acid, which means that it goes through two different equilibria before being completely reduced. We also know, from experience, that sulfuric acid is a very strong acid. Strong acids are characterized by their ability to dissociate, resulting in the loss of a proton. As such, the first equilibrium, when sulfuric acid loses one hydrogen to form its conjugate base HSO4–, is heavily favored towards the formation of products. We would expect, therefore, that in this solution, there would be more products than reactants—more HSO4– and H+ than H2SO4. This indicates that neither HSO4– nor H+ can be the correct answer; we are looking for the species that is present in the smallest concentration. Knowing this, we can eliminate choices (C) and (D). Additionally, the HSO4– can undergo further dissociation to lose yet another hydrogen atom to form the sulfate anion (SO42–). In this equilibrium, HSO4– acts as the acid (Lewis acid—electron acceptor) and SO42– is the conjugate base (Lewis base—electron donor). HSO4– is an amphoteric species because it can act as either a base or an acid, depending on whether it loses or gains electrons. Since the starting reactant, sulfuric acid, is a strong acid, its conjugate base, HSO4–, must be a weaker amphoteric species. This indicates that the second deprotonation will occur less strongly. While H2SO4 will easily dissociate into HSO4–, only a very small amount of HSO4– will dissociate into SO42–. Also, in our situation, with a very small percentage of water (2%), not all of the sulfuric

26

Physical Sciences

acid will be able to dissociate—the vast majority of it will remain undissociated in solution. Among all the species present, the one that will be present in the smallest amount will be SO42–—choice (A) is the correct answer. (B) Distortion. This will be the most predominant species in the solution. (C) Distortion. This species will be more predominant than SO42–. (D) Distortion. This species will be more predominant than SO42–.

Discrete Set III (Items 48–52) 48. (B) Glancing at the answer choices, we can categorize them into two major categories; our first task is to determine if the Zn will dissolve. Since the electrons are on the left side of the reaction, this is a reduction half reaction where the oxidation state of the zinc cation is reduced to zero in the zinc solid. The item stem, however, asks us to assess the reverse reaction—that in which solid zinc dissolves into solution. We are provided with the standard potential for the given reaction (–0.763 V). We can infer from this information that the standard potential for the reverse reaction is +0.763 V. Thus, the reverse reaction is an oxidation half reaction that takes zinc solid (oxidation state of zero) to zinc cation (+2 oxidation state). Since the standard potential for this oxidation is positive, we conclude that the oxidation of zinc is spontaneous—the first portion of the correct answer should state that the zinc will dissolve into solution, that is, it will be oxidized to Zn2+. Knowing this, we can eliminate choices (C) and (D). Our final task is to determine the products of the reaction. Since the half reaction states that Zn is oxidized to Zn2+, we expect the neutral product to be ZnCl2 (the chloride ion will always have a charge of –1). Only choice (B) matches this prediction, and is thus the correct response. (A) Distortion. Since the zinc cation has an oxidation state of 2+ and chloride will always have a charge of 1–, the product will be ZnCl2. (C) Opposite. Zinc solid will be oxidized to Zn2+ because the oxidation half reaction has a positive oxidation potential. (D) Opposite. Zinc solid will be oxidized to Zn2+ because the oxidation half reaction has a positive oxidation potential. 49. (D) This is a relatively straightforward item in which we are tested on the definition of power. The answer choices present us with various combinations of variables, but only one of them will allow us to calculate the power of a car. What is power? Power is the amount of energy expended over a given time—it has the units of joules/second, or watts. The only answer choice that provides energy and time is choice (D), the correct answer. (A) Distortion. This does not provide enough information to calculate power. (B) Distortion. This does not provide enough information to calculate power. (C) Distortion. This does not provide enough information to calculate power. 50. (B) The item stem provides us with a sketch of a molecule and asks us to determine the longest bond. The answer choices make reference to the different bonds within the molecule. What determines bond length? Two factors affect the bond length: atomic radius and number of bonds. Atomic radius dictates how far apart two atoms will be

27

AAMC Test 3 Explanations

in a bond (atomic radius is measured from the center of each atom to the farthest orbiting electron shell), while the number of bonds dictates bond strength—the stronger the bond, the shorter the bond between atoms. From our knowledge of periodic trends, we know that the atomic radius decreases as we travel across a period and increases as we travel down a group. Referring to the periodic table, we notice that hydrogen is in the first period, and thus a very small element, while nitrogen and oxygen are in the second period. Therefore, among the three types of atoms present, the largest atoms are those of nitrogen and oxygen—hydrogen is considerably smaller. Due to its smaller radius, any bond with hydrogen will also be shorter—we can eliminate choice (A). To decide between the remaining bonds, we must now analyze the bond strength. The Lewis structure of the molecule is:

O–

O –

O

N+

O

H

N+ O

O

H

Analyzing the three bonds between oxygen and nitrogen, we notice that bonds C and D are the hybrid of a single and double bond, due to resonance. The greater the number of bonds between atoms, the stronger the bonds, resulting in shorter bond length. Since the bond between nitrogen and the alcohol oxygen is only a single bond, we expect it to be longer than the other nitrogen-oxygen bonds—choice (B) is the correct answer. (A) Opposite. This bond will be shorter than bond B because hydrogen has a very small radius. (C) Opposite. This bond will be shorter than bond B because it is a hybrid of a single and double bond. (D) Opposite. This bond will be shorter than bond B because it is a hybrid of a single and double bond. 51. (A) This item may appear complex upon first glance. However, the underlying concept that it tests is the photoelectric effect. We are told that a photon strikes an atom and an electron is ejected. We are given the energy of the photon and ionization energy of the electron (the energy required to remove one electron), and are asked to calculate the kinetic energy of the ejected electron. The photoelectric effect describes the conservation of energy at an atomic level—between the energy of the incoming photon and the kinetic energy of the electron. The kinetic energy of the electron can be calculated using the equation: KE = E – φ where KE is the kinetic energy, φ is the work function of the atom (in this case, the ionization energy), and E is the energy of the incoming photon. In other words, part of the energy provided by the photon is used to remove the electron from its orbital, whereas the remaining energy is converted directly into the kinetic energy of the electron. Using the values provided in the question stem, we obtain the kinetic energy of the electron:

KE = 15 eV – 13.6 eV



KE = 1.4 eV

Choice (A) is the correct answer. (B) Distortion. This is the energy required to remove the electron from the hydrogen atom. (C) Distortion. This is the energy of the incoming photon—some of this energy is spent in liberating the electron from the hydrogen atom’s orbit.

28

Physical Sciences

(D) Distortion. This answer would be incorrectly obtained if you added, instead of subtracted, the energy of the photon and the work function. 52. (B) The item stem mentions the radioactive decay of a certain atom and the answer choices offer four possible products of this decay. To solve this question, we must first have a good understanding of β+ decay, or electron capture. During this process, one positron is emitted from the atom. A positron is a particle that has the same mass as an electron but a positive +1 charge. Because of its positive nature, the release of a positron during β+ decay means that the resulting atom will have one less proton, since the charge of the reaction must be conserved. During beta plus decay, a proton decays into a neutron and positron. The positron is emitted, whereas the resulting neutron remains in the nucleus. As a result of this reaction, the number of protons (atomic number) has decreased, while the number of nucleons (mass number) has remained the same. Thus, the radioactive decay is: 7 7   ​  ​Be →   ​  ​X + β+ 4 3 In this reaction, both charge and mass are conserved—choice (B) is the correct answer. (A) Distortion. The mass number does not change because the loss of a proton was offset by the gain of a neutron. (C) Distortion. In beta plus decay, there is loss of a proton, decreasing the atomic number—this nucleus would result from a gamma decay. (D) Distortion. In beta plus decay, there is loss of a proton, decreasing the atomic number, while the mass number remains unchanged.

29

Verbal Reasoning

Verbal reasoning ANSWER KEY 53. D

64. B

75. A

86. A

54. C

65. A

76. D

87. D

55. C

66. D

77. A

88. A

56. B

67. A

78. C

89. B

57. A

68. A

79. B

90. C

58. B

69. C

80. D

91. D

59. C

70. C

81. A

92. B

60. D

71. B

82. A

61. B

72. A

83. B

62. C

73. D

84. A

63. C

74. B

85. A

31

AAMC Test 3 Explanations

AAMC Test 3 Verbal Reasoning—Kaplan Explanations Passage I (Items 53–57) Passage Map Passage Type: Social Sciences Topic: The drug problem Scope: What to do about it Purpose: To advocate the legalization of drugs Mapping the Passage: ¶1: Drug problem getting worse – what to do? ¶2: One solution – legalize drugs ¶3: 3 reasons to legalize drugs ¶4: Increased drug enforcement isn’t working. ¶5: Drug enforcement is costly. ¶6: Drug enforcement diverts key resources and prison space. ¶7: Connection of drugs/crime would decrease if drugs legalized. ¶8: If drugs cheaper, addicts wouldn’t commit as many crimes to pay for them. ¶9: Legalization would lead to less corruption in law enforcement. ¶10: Enforcing drug laws regulates illegal market, but doesn’t protect victims—analogy to prohibition. ¶11: Advantages of repealing drug laws: new tax revenue, decreased crime, better quality of life ¶12: Two assumptions why people wouldn’t use drugs even if legal ¶13: Legalization would work, but idea usually just dismissed. 53. (D) This incorporation item quotes an outside authority who clearly thinks that cocaine and nicotine are dangerous. As a “which of the following” type of question, it is difficult to formulate a precise prediction. Instead, we’ll need to make sure that we select the answer choice that fits with the information provided in the question stem and the passage. How does this authority feel? All we know from the item stem is that this person believes that cocaine and nicotine are destructive. Since we have no evidence that would suggest her stance on legalization/ abolition, we can safely eliminate choices (A) and (B). Of the remaining answer choices, only choice (D) reflects our understanding of this authority’s stance on drugs—that while both drugs are destructive, cocaine is more so. Choice (D) is thus the correct answer. (A) Out of scope. We know nothing about this authority’s views on drug legalization. (B) Out of scope. We know nothing about this authority’s views on drug legalization. (C) Opposite. As outlined in the item stem, the authority definitively believes that these drugs are dangerous.

32

Verbal Reasoning

54. (C) What is the author’s suggestion regarding the legality of drugs? Since the purpose of this passage, as indicated in our passage map, is to advocate the legalization of drugs, the author would support any federal law that reduces the circumstances that promote “drug problems.” Indeed, the author will agree with any answer choice that reflects legalization of drugs as a means of solving the problem. Choice (C) is the only answer choice that reflects this tone. As stated in paragraph 11, by encouraging safe, legal usage of drugs, the dangerous, illicit market will no longer be reinforced. Choice (C) is thus the correct answer. (A) Opposite. According to paragraph 4, the recent drug-enforcement efforts have not improved the “drug problem.” As such, increased measures will not be expected to improve the situation. (B) Opposite. In paragraph 11, the author suggests that the sales of drugs would lead to a substantial tax benefit for the country. Therefore, the author would not suggest tax-exempt status for the newly legal drug trade. (D) Opposite. According to paragraph 4, the recent drug-enforcement efforts have not improved the “drug problem.” As such, increased measures will not be expected to improve the situation. 55. (C) Since this is a Roman numeral question, a good strategy is to first identify which statement appears most often in the answer choices—if this statement is not correct, many answer choices can be eliminated. Statement I appears most, but since it appears in all of the answer choices, there is no reason to analyze it—it must be true. Next, since statements II and III each appear twice, we can start with either of them—we’ll start with statement II. First, though, let’s get a bearing on which part of the passage we need to reference. The item stem asks us to identify the explicitly mentioned claims that justify the assertion that treasuries would enjoy a benefit as a result of the legalization process. Where is this discussed in the passage? According to our passage map, paragraph 11 addresses the advantages of repealing drug-prohibition. Specifically, it cites “reduced government expenditures on enforcing drug laws and new tax revenue” as reasons why “public treasuries would enjoy a net benefit.” Let’s now see which of the two remaining statements matches this criterion. Statement II is not mentioned in this part of the passage, so it is false, allowing us to eliminate choices (B) and (D). Statement III, however, states fewer public funds being spent—this is an advantage that is explicitly mentioned in the passage. As such, statements I and III are true, and choice (C) is the correct answer. Roman numeral I – True: Tax revenue is mentioned explicitly in paragraph 11 as an advantage of drug legalization. Roman numeral II – False: This is not mentioned in paragraph 11 as an advantage of drug legalization. Roman numeral III – True: Reduced government expenditures is mentioned explicitly in paragraph 11 as an advantage of drug legalization. (A) Distortion. Statement III is also correct. (B) Distortion. Statement II is false. (D) Distortion. Statement II is false. 56. (B) Where does the author make this argument? According to our passage map, the author provides two assumptions in paragraph 12 as to why people wouldn’t use drugs, even if legalized. Here, the author asserts that “most illegal drugs are not so dangerous as is commonly believed, and that the drugs and methods of consumption that are most risky are likely to prove unappealing to many people, precisely because they are so obviously

33

AAMC Test 3 Explanations

dangerous.” The correct answer will be the answer choice that provides further evidence to prove that either of these assumptions is true. Of the given answer choices, only the finding described in choice (B) validates the author’s assumptions, and is, therefore, the correct answer. (A) Faulty use of detail. In paragraph 3, the author notes that most Americans are hostile to legalizing drugs, but this fact refers to why people wouldn’t support changes in the law, not why they wouldn’t use drugs themselves. (C) Out of scope. The author doesn’t tell us anything regarding the thought process of people who consider using drugs. (D) Opposite. This would weaken the author’s claims that there wouldn’t be a dramatic increase in drug abuse. 57. (A) Where does the author make an assertion regarding the street price of illegal drugs? According to our passage map, paragraphs 7 and 8 argue that the lower costs of drugs as a result of legalization would lead to a decrease in the number of crimes committed. Implicit in this claim is the idea that the high price of drugs, due to their illegality, has led to a certain number of crimes committed in order to pay for the addictive substances. As such, choice (A) correctly addresses the relationship between enforcement and drug price—as drug enforcement has been high (due to the illegality of the substance), the street price of the drugs has also been high. (B) Opposite. The street price would go up if the laws were more stringent. (C) Opposite. In paragraph 8, the author suggests that legalization would lead to lower street prices. (D) Opposite. Paragraph 8 suggests that the price of drugs would be “significantly cheaper” if drug use was legalized, suggesting a clear relationship between drug enforcement and the street price of drugs.

34

Verbal Reasoning

Passage II (Items 58–62) Passage Map Passage Type: Natural Sciences Topic: Wolves Scope: What we imagine them to be Purpose: To argue that we imagine much of what we think we know about wolves Mapping the Passage: ¶1: We don’t know a lot about wolves—we often make up what we know. ¶2: Nunamiut imagine wolves to be like people. ¶3: Facts regarding wolves: hunting, playing, communicating, etc. ¶4: It can be hard to see the wolves in their habitat. ¶5: Wolves trigger human imagination – e.g. Bella Coola, wolf-haters, wolf-lovers. ¶6: Most of what we think we know is just opinion. ¶7: Wolves differ like clouds do. ¶8: Author’s experiences with wolves ¶9: Campbell: we create our own ideas about animals. 58. (B) Where does the author mention the Bella Coola Indians? According to our passage map, the author makes this reference in paragraph 5, as evidence that “the wolf exerts a powerful influence on the human imagination.” Indeed, the correct answer will address the author’s reason for citing this evidence—to support the stated conclusion. From the given answer choices, only choice (B) mirrors this reasoning. The author mentions the Bella Coola Indian belief so as to provide evidence for her conclusion that the wolf provokes fanciful thinking in humans. Therefore, choice (B) is the correct answer. (A) Faulty use of detail. Although the author would agree that we know little about the wolf, the author cites this legend as direct evidence of the effect the wolf has on human imagination (paragraph 5). (C) Out of scope. The author never makes the argument that people have always revered the wolf. (D) Faulty use of detail. The Bella Coola story is an example of a myth that indicates that the wolf has had a powerful influence on human imagination. However, the author never argues that the wolf is humanlike. 59. (C) A quick scan of the passage shows us that the word “rigorous” is located in paragraph 7. Here, the author argues that, like clouds, we cannot be rigorous about wolves. The meaning of this word, rigorous, must apply to both wolf and cloud, and must agree with the tone of the passage. According to our passage map, the author’s purpose is to maintain that we imagine much of what we think we know about wolves. Like clouds, wolves are an elusive, hard to define phenomenon. The only answer choice that fits this prediction is choice (C). We cannot be precise about clouds or wolves—we know little about either, as their natures, according to the author, are difficult to grasp. As such, the only context that applies to both wolves and clouds is the one presented in choice (C), the correct answer.

35

AAMC Test 3 Explanations

(A) Out of scope. The author makes the comparison between clouds and wolves in that their natures are illusive—one cannot be exact (precise) in understanding or describing either. (B) Out of scope. The author makes the comparison between clouds and wolves in that their natures are illusive—one cannot be exact (precise) in understanding or describing either. (D) Out of scope. The author makes the comparison between clouds and wolves in that their natures are illusive—one cannot be exact (precise) in understanding or describing either. 60. (D) As noted in the item stem, the author asserts that we imagine, or create, what we know about the wolf, rather than base our thinking on concrete facts. Indeed, this is the recurrent theme found throughout the passage. In order to weaken this argument, we need to show that the opposite is true—that what we know about the wolf is not imagined at all, but rather based on observed and established facts. In asserting that scientists have produced a wealth of knowledge regarding the wolf, choice (D) is the only answer choice which upends the author’s mystical, elusive description of wolves. (A) Faulty use of detail. Whether this incident happened has no bearing on the author’s conclusion regarding the elusive nature of wolves. (B) Opposite. This statement would further support the author’s contention that wolves are less known than created by us (and our imaginations). (C) Out of scope. Regardless if these behaviors are instinctive, this statement still doesn’t help humans to know the wolf. 61. (B) What do we know about the nature of wolves and what would the author suggest to the historians? Since the item stem focuses on approaching the wolf, it can be inferred that the author’s advice hinges on her personal convictions regarding the danger of wolves. According to paragraph 5, “wolf-haters want to say they are born killers, which isn’t true … [while] wolf-lovers want to say no healthy wolf ever killed anyone in North America, which isn’t true either.” Knowing this, we can expect the author to advise some reservation while approaching the wolves, since they can be dangerous. The only answer choice that reflects this prediction is choice (B), the correct answer. (A) Opposite. We learn in paragraph 5 that wolves “have killed Indians and Eskimos.” Therefore, the author would advise the historians to approach with caution. (C) Opposite. We learn in paragraph 5 that wolves “have killed Indians and Eskimos.” Therefore, the author would advise the historians to approach with caution. However, there is no indication of scorn or contempt regarding wolves—or that they are “quite” dangerous, only that they have been known to kill. (D) Opposite. We cannot be confident about the wolf because we don’t really know any more about it than we did in the past—this is the author’s assertion throughout the passage, the elusive nature of the wolf. 62. (C) How does this new information relate to that already presented in the passage? Throughout the passage, the author cites numerous opinions regarding the wolf. In paragraph 2, we are privy to the ideas of the Nunamiut Eskimos. In paragraph 5, we hear the beliefs held by the Bella Coola Indians, as well as wolf-haters and wolflovers. Furthermore, in paragraph 8, we read about the author’s personal experiences with learning about the wolf. What does this culminate in? After presenting each of these examples, the author concludes, in paragraph

36

Verbal Reasoning

9, that what we know about the wolf is less known than created—we imagine what we want the wolf to be. How does the World Book Encyclopedia apply to this passage? It presents, just like the many other cited examples, a created and imagined definition of what the wolf “is.” As such, it would be included as an additional example that what we know of the wolf is imagined. Therefore, choice (C) is the correct answer. (A) Out of scope. The author never asserts that wolves are “excellent hunters.” (B) Faulty use of detail. This quote makes no assertion that people see wolves as being like themselves. In fact, the passage mentions that wolves’ behavior resembles that of a dog, not of a human. (D) Opposite. The author provides no evidence that would suggest such a harsh and degrading opinion of wolves. The passage implies that the author feels respect and intrigue towards this animal.

Passage III (Items 63–67) Passage Map Passage Type: Humanities Topic: Art Scope: Art traditions and class experiences Purpose: To argue that artists, limited by established conventions, find difficulty in applying their style to other subjects Mapping the Passage: ¶1: Artists used conventions that corresponded to the ruling class for whom they painted—they considered this depiction as preserving truths, rather than artistic conventions. ¶2: 19th c., some artists tried to depict the experiences of other classes – hard to do because the conventions used only seemed to accurately portray the ruling class; e.g. Brown: Work. ¶3: 20th c., old ways dismantled; tried to show experience of other classes, only resulted in painting about isolation or the painting itself (abstract art) ¶4: New freedom not used b/c painters were still being trained in old conventions ¶5: This example shows the uncertainty of an art that, limited by its narrow conventions, may claim to be universal, when, in reality, is not 63. (C) This is a global item. Fortunately, we have already laid the groundwork to answer this question by creating our passage map; specifically, by recording the purpose of the passage. According to our map, the author wrote this article to argue that artists, limited by established conventions, find difficulty in applying their style to other subjects. The correct answer should reflect this theme. From the given answer choices, only choice (C) reflects this purpose, and is therefore the correct answer. (A) Faulty use of detail. Although the author mentions this explicitly in the first paragraph, this sentence is introductory, and not descriptive of the overall purpose of the passage. (B) Faulty use of detail. Although the discussion of professional training in the conventions of art is part of this passage, this doesn’t reflect the author’s overall purpose.

37

AAMC Test 3 Explanations

(D) Out of scope. It’s true that those who abandoned conventional methods had a hard time (e.g., Ford Madox Brown) but not because of opposition from the ruling classes. 64. (B) A quick scan of the passage locates the word “tradition” in the beginning of the second paragraph. Here, “certain artists…[try] to extend the professional tradition of painting.” What tradition of painting was discussed in paragraph 1? According to our passage map, these artists used conventions that corresponded to the ruling class for whom they painted. Thus, we can expect this term, tradition, to relate to this definition, making choice (B) the correct answer. (A) Out of scope. The author never asserts that such conventions were the best way to record and preserve eternal artistic truths, rather, that the artists felt that these were the only ways to do so. (C) Opposite. This reference to symbolism is made in paragraph 2, in relation to painters like Brown, who were trying to break away from tradition. (D) Opposite. The discussion of the unconscious occurs when the author is talking about the 20th century, when traditions were “dismantled.” 65. (A) Before we can decide what is implied by the passage, we need to review the main argument. According to our passage map, the author wrote this passage to argue that artists, limited by established conventions, find difficulty in applying their style to other subjects. The passage describes the struggle to apply the artists’ learned conventions to other social classes, only to find that such application is difficult, if not impossible. It can be inferred, then, that art is at its best (i.e., art is universal) when it transcends convention. Based on this, we can see that choice (A) is the correct answer. (B) Out of scope. Although the artists described in paragraph 1 believed that their art preserved eternal truths, the author argues that such a view was limited, in that such conventions did not aptly portray other classes (paragraph 2). (C) Opposite. The limited conventions utilized by the artists described in paragraph 1 did indeed correspond to the social manners of the ruling class. Yet, these same conventions failed to capture the life of the working class (paragraph 2). Therefore, art is not at its best (not universal). (D) Out of scope. There is no indication that art is at its best in treating religious themes. 66. (D) Where do we find this argument in the passage? In paragraph 1, we read that “these conventions corresponded so closely to the…social manners…of the class [the artist] was serving.” What conventions does this statement refer to? It cites the conventions taught to the artist while in training to work for the ruling class. Thus, these conventions corresponded to the social manners of the ruling class, ultimately because the ruling class hired the artists to paint. With this prediction in mind, we can safely select choice (D) as the correct answer. (A) Out of scope. The author makes no such argument in the passage. (B) Out of scope. It is never mentioned that the artists felt that these were the only experiences worth recording— rather, it was the ruling class which encouraged such conventions through their patronage. (C) Out of scope. There is no evidence that would suggest that artists aspired to be of the ruling class.

38

Verbal Reasoning

67. (A) Where does the author mention Brown’s painting? Our passage map directs us to paragraph 2, where the author argues that during the 19th century, some artists tried to paint the experience of other classes, but found such a task difficult because the conventions used only seemed to accurately portray the ruling class. Indeed, it was difficult to expand their subject matter to portray the experiences of other classes. As such, choice (A) is the correct answer. (B) Out of scope. The reference to Brown’s painting has nothing to do with how powerful realistic art is. In fact, the author doesn’t even consider Brown’s painting to be realistic: he calls it “mythological or symbolic.” (C) Out of scope. Although Brown’s painting is an example of the problems inherent in expanding the subject matter of art, it doesn’t prove that such problems are with us today. In fact, the passage does not discuss today’s situation. (D) Faulty use of detail. Although this statement is made in paragraph 1, it is not related to the mention of Brown’s painting in paragraph 2.

Passage IV (Items 68–74) Passage Map Passage Type: Natural Sciences Topic: Earth’s interior Scope: Earth’s core is cooling Purpose: To explain why and how the Earth’s core is cooling Mapping the Passage: ¶1: How earth’s interior formed; core of planet is cooling ¶2: Heat released from decay of radioactive elements augments the cooling effect; most heat that originates in mantle, accounts for surface geological activity ¶3: Core made mostly of iron but other things as well; once completely molten, now inner part is partially solid b/c of pressure ¶4: Best evidence of core cooling is magnetic field, which is the product of electric currents; Dynamo theory ¶5: Dynamo device explained 68. (A) Since this is a detail item, we must reference the passage to decide which answer choice is not mentioned as a contributing component to the internal heat of the earth. Since the formation of the core is mentioned in paragraph 1, we’ll start our search there. In paragraph 1, we find that, during the formation of the earth, enough gravitational energy was released to melt the earthly core. “As the heavier materials separated from the lighter ones, additional energy was released, adding to the earth’s internal heat.” Based on this, we can safely eliminate choices (B) and (D), since they directly contributed to the initial internal heat of the earth. As we continue our search into paragraph 2, we read that, while the earth is “shedding its primordial heat” as it cools, “the earth [also] casts off the heat generated by the decay of radioactive elements trapped in its interior.” As such, we can now eliminate choice (C) as well. Fluid motions in the outer core, which are mentioned in paragraph 4, are the

39

AAMC Test 3 Explanations

cause of the magnetic fields, and are not responsible for the internal heat of the earth. Indeed, choice (A) is the correct answer. (B) Opposite. Primordial gravitational processes are mentioned in paragraph 1 in conjunction with the initial formation of the earth’s core—it led to establishment of the initial internal heat. (C) Opposite. According to paragraph 2, “the earth casts off the heat generated by the decay of radioactive elements trapped in its interior,” indicating that such decay contributes to the internal heat of the earth. (D) Opposite. According to paragraph 1, “as the heavier materials separated from the lighter ones, additional energy was released, adding to the earth’s internal heat.” 69. (C) Where do we read about terawatts in the passage? A quick passage scan reveals that we can find the necessary information in paragraph 2. Here, the author informs us that 42 terawatts of heat continuously escape from the earth’s surface. Yet, the item asks us to provide the amount of heat provided by the sun, not the earth’s surface. If we read an additional sentence, we find that this value, 42 terawatts, is “only about a thousandth of the heat provided by the sun.” As such, we would expect the sun to provide 42 terawatts multiplied by a thousand, or 42,000 terawatts. We can match our prediction with choice (C), the correct answer. (A) Faulty use of detail. This is the amount of energy lost by the earth’s surface. (B) Distortion. This is a tenth of the amount of heat provided by the sun. (D) Faulty use of detail. This is the amount of watts that escape from the surface of the earth. 70. (C) Where does the author discuss the earth’s core composition? According to our passage, map, this discussion is located in paragraph 3. Here, we find that earth’s inner core is ​ _13 ​solid, whereas the other ​ _23 ​is molten. Additionally, we know that the core is made up mostly of iron—which agrees with the idea presented in paragraph 1 that the “metallic constituents collected at the center” of the earth during formation. The only answer choice that agrees with our findings is choice (C), the correct answer. (A) Opposite. The core is not predominantly solid, it is mostly fluid. Regarding radioactivity, we do not know how much of the core is radioactive, so we cannot assume that it is predominant. (B) Faulty use of detail. There is no mention of the core being made up of gases—only that gases were involved in the initial formation of the earth. Also, the earth has a magnetic field, but this is not considered to be part of the earth’s core. (D) Opposite. The core is not predominantly solid, it is mostly fluid. Also, the earth has a magnetic field, but this is not considered to be part of the earth’s core. 71. (B) Where does the author discuss the magnetic field? Our passage map leads us to paragraph 4, where we learn that “magnetized rock show(s) that the earth has had a magnetic field for at least three billion years.” Thus, magnetic data obtained from rocks is the best way to show the continuous existence of earth’s magnetic field. Based on this information, choice (B) is the correct answer. (A) Faulty use of detail. Geological activity at the earth’s surface is the result of heat released from the core, not the magnetic field.

40

Verbal Reasoning

(C) Faulty use of detail. Conductivity is used to explain how a dynamo works, not as evidence of the earth’s magnetic field. (D) Faulty use of detail. This conversion of energy is used to explain how a dynamo works; it is not used as evidence of the earth’s magnetic field. 72. (A) Where does the author mention the dynamo theory? According to our passage map, we need to return to paragraph 4. Here, we read that “the only theory that can explain the persistence of the (magnetic) field and its propensity for reversing itself is the dynamo theory, which holds that the magnetic field results from fluid motions in the outer core.” Since only the dynamo theory can account for the persistence of the magnetic field, and its tendency to reverse, choice (A) is the correct answer. (B) Out of scope. The strength of the earth’s magnetic field is not mentioned in the passage. (C) Opposite. According to paragraph 4, the magnetic field is “the clearest evidence that the core is cooling,” not heating. (D) Faulty use of detail. The dynamo theory accounts for the magnetic field, not the electrical conductivity in the inner core. 73. (D) Where does the author discuss magnetic fields? Our passage map leads us back to paragraph 4—where we find that “the magnetic field results from fluid motions in the outer core.” Only choice (D) matches the assertion made in the passage and is thus the correct answer. (A) Faulty use of detail. Fluid motion is related to the dynamo theory, which explains how magnetic fields work, but not how they originate. (B) Out of scope. The rotation of the earth is not mentioned in the passage. (C) Faulty use of detail. Mechanical energy is discussed in conjunction with the operations of a dynamo device, not as an explanation for how magnetic fields are formed. 74. (B) How are these disciplines interwoven into the passage? The ideas of geology, physics, chemistry, mathematics, and other disciplines were cited so as to better understand why and how the Earth’s core is cooling. This is accomplished by integrating these studies so as to form hypotheses regarding the birth of important, earth-shaping phenomenon. Thus, this integration would be most akin to any study that also compiles various disciplines so as to understand the past. Of the given answer choices, only choice (B) assimilates several areas of study so as to produce theories regarding the past, and is, therefore, the correct answer. (A) Opposite. The theory of how artistic inspiration is achieved relies solely on the field of art, not various, cumulative fields. (C) Opposite. This does not require the cumulative effort of several disciplines. (D) Opposite. This does not require the cumulative effort of several disciplines.

41

AAMC Test 3 Explanations

Passage V (Items 75–79) Passage Map Passage Type: Social Sciences Topic: TV Scope: Dynamics of television shows Purpose: To argue that TV purposely uses imprecise language, shielded passion, and its own version of reality so as to encourage its viewers to “stay tuned” Mapping the Passage: ¶1: TV, to ensure its own survival, makes its audience passively captive; Malcolm Muggeridge ¶2: TV has no distinct vocabulary—it recycles ideas and characters. ¶3: Regularity of a series gives us a way to measure our lives—TV portrayals set behavioral expectations for viewers. ¶4: TV is undemanding—it lacks extreme passion so as to make a “good program,” ensuring that it is not turned off. 75. (A) Glancing through the passage, we notice that newscasters are mentioned in paragraph 1. Here, the author asserts that, for newscasters, “’stay tuned’ is the eleventh and commanding commandment.” Thus, television newscasters attempt to keep their viewers watching—they encourage them to stay passive, as any encouraged active response would cause the audience to miss the next program.” The answer that matches our line of reasoning is choice (A), the correct answer. (B) Opposite. In paragraph 1, the author tells us that, while the viewer becomes emotional, “catharsis does not follow.” (C) Opposite. In paragraph 1, the author asserts that “if we are moved to leave the viewing chair we may miss the next program.” Thus, the newscasters try to keep their audience “tuned” in. (D) Faulty use of detail. In paragraph 1, we read that the “segmentation and ‘personalization’ of the ‘news’ actually confuse us.” Yet, this refers to the effect of the news, not the newscaster, as is later mentioned. 76. (D) Why might such a real-life argument not make for good television? According to our passage map, “good program(s)” are discussed in the last paragraph. Here, we read that such programming is void of rich language, heightened passions or obsessions. Thus, we can expect a real-life argument not to be good television if it violates any of these moderated characteristics. Indeed, if such an argument were too passionate, it “might cause (the) audience to switch off.” Our prediction matches nicely with choice (D), the correct answer. (A) Out of scope. There is nothing in the passage that suggests that a real-life argument would be any more or less convincing than that between television actors. (B) Opposite. The author tells us that we’re drawn to TV because it is inarticulate: “A lack of articulacy is the badge of sincerity…” (paragraph 1). Indeed, it is the “good program” that is “docked of amateurish or embarrassing passions.” Therefore, we can expect those in a real-life argument to be more capable of articulating their emotions. (C) Out of scope. The author argues that the audience, not the television characters, is passive.

42

Verbal Reasoning

77. (A) A brief skim of the passage reveals that television characters are discussed in paragraph 1, when the author states that “’characters’ we have seen before, advertisements we ‘love’—may well be the evidence that originality has scant future on the box.” The author cites these characters and advertisements as evidence for the conclusion that originality doesn’t belong in television. Therefore, choice (A) is the correct answer. (B) Out of Scope. The author doesn’t mention or allude to other programs which have a numbing effect on viewers. In paragraph 1; Malcolm Muggeridge only attributes such effects to the news. (C) Faulty use of detail. Although the author certainly makes this claim (paragraph 1), the evidence regarding television characters is not mentioned to support it. (D) Faulty use of detail. Although the author’s overall argument points to this claim, the evidence regarding television characters is mentioned explicitly to show that originality doesn’t belong in television. 78. (C) To answer this question, we need to first identify the author’s argument. The author’s argument can most easily be found by referring to our passage map. Here, we noted that the author wrote this passage to argue that TV purposely uses imprecise language, shielded passion, and its own version of reality so as to encourage its viewers to “stay tuned.” Any evidence that would suggest otherwise would most strongly challenge the passage information. If a television character was found to be sympathetic, passionate, and articulate, this would clash with the author’s assertions that “good” television “has no distinct vocabulary,” and is void of “embarrassing passions”—making choice (C) the correct answer. (A) Out of Scope. Since the passage doesn’t mention the term “critic,” we can make no decision on how this statement would affect the author’s main argument. (B) Opposite. This answer mirrors what we have already read regarding television programs—simple language and images which encourage a captivated, yet passive audience. This evidence would strengthen the author’s claims. (D) Opposite. Paragraph 2 argues that “television is a voracious recycler and mixer of a confluence of concepts.” Thus, any evidence showing that a television series exhibited these traits would strengthen the author’s argument. 79. (B) Where does the author mention what a “good program” is? According to our passage map, “good program[s]” are mentioned in the last paragraph. Here, we read that such programming is void of rich language, heightened passions or obsessions that “might cause [the] audience to switch off.” Only choice (B) mirrors the ideals of a “good program”; it involves the viewers just enough so as to keep them watching. (A) Opposite. In the first paragraph, the author argues that viewers have an emotional response to what they see, but are only stimulated enough so that an active response does not follow. Thus, viewers are not made to feel the full impact of their emotions; they are restrained enough so as to guarantee continued television watching. (C) Opposite. Paragraph 4 argues that “good” programming does not feature passionate or articulate characters. (D) Opposite. Since paragraph 1 argues that “originality has [a] scant future” in television, it is reasonable to conclude that interesting/unusual topics will not be a part of “good” programming.

43

AAMC Test 3 Explanations

Passage VI (Items 80–85) Passage Map Passage Type: Humanities Topic: Confucius and li Scope: What is li and what it signifies Purpose: To describe and support li as a ceremony of life Mapping the Passage: ¶1: Confucius: human powers have magical quality – path to right way of existence is li ¶2: Li is about tradition and convention – Confucius taught a need to act according to and in submission to li. ¶3: Li is invisible and intangible, rooted in dignity; example of threatening leader vs. one embracing li ¶4: Li also has physical powers; example of moving a book ¶5: Austin: performative utterances – ritual word is an act of action ¶6: Example of performative utterance – wording of a will ¶7: Austin: substance of human existence is ceremony – ritual words are ceremony 80. (D) Since this is a global question, we can expect many of the answer choices to offer supportive details for the purpose of the passage; the answers too refined in their scope can be eliminated. To make an accurate prediction, we need to establish the author’s overall intentions. As such, the author’s central thesis can be best found by reviewing the passage map. We previously noted that the author wrote this passage to describe and support li as a ceremony of life. From the given answer choices, choice (D) is a simple restatement of this main theme, making it the correct answer. (A) Faulty use of detail. Although the author mentions this in the last paragraph, this is not the main focus of the entire passage. (B) Faulty use of detail. Although the author cites an example in which li can be used to get what one wants, this is not the main purpose of the passage. (C) Out of scope. There is no information in the passage that suggests that performative utterances are more honest. 81. (A) Since Confucius can only endorse what is alluded to in the passage, the correct answer must have concrete evidence that suggests such action. According to our passage map, paragraph 3 provides an example in which li is endorsed. Here, we learn that “li works through spontaneous coordination rooted in reverent dignity” and should be used in lieu of coercion. Only choice (A) matches this rationale, and is therefore the correct answer. (B) Opposite. As our passage map indicates, li works invisibly and intangibly, but through ceremony and dignity, not through magical powers. (C) Opposite. According to paragraph 2, “one has to labor long and hard to learn li.” This implies that it requires quite a bit of effort to learn li, making this answer choice incorrect. (D) Out of scope. There is nothing mentioned in the passage regarding the avoidance of risks.

44

Verbal Reasoning

82. (A) Where does Confucius use the term “magical?” A quick passage scan reveals its location in paragraph 1—here, the author states that “Confucius saw…that the truly, distinctively human powers have…a magical quality.” Why magical? The author continues, saying that Confucius sought to “reveal what is already so familiar and universal.” Thus, this “magical” designation is made so as to highlight the not-so-ordinary potential of familiar and normal conventions. Knowing this, only choice (A) can be the correct answer. (B) Faulty use of detail. This is the “common definition” trap. Li is not magic in the sense of magic act entertainment, since it has nothing to do with the supernatural. (C) Out of scope. The author never mentions Confucian masters in the passage. (D) Faulty use of detail. The author uses the example of moving the book in paragraph 4 as a ceremonial statement (used by the teacher so as to get the student to obtain the book for him). 83. (B) This is a Roman numeral question; a good strategy is to first identify which statement appears most often in the answer choices—if this statement is not correct, many answer choices can be eliminated. In our case, each of the statements appears twice, so we’ll start with statement I. According to our passage map, performative utterances are mentioned in paragraph 5—they are “statements we make which function…like the ‘operative’ clause in a legal instrument…They are the…execution of the act itself.” Paragraph 5 then provides an example of a will, in which a performative utterance (giving the watch to a brother) is demonstrated. The simple act of promising the watch to the brother ensures that the act of giving actually occurs. Statement I, a plea for help, does not fit the definition of a performance utterance. As such, we can safely eliminate choices (A) and (C). Of the remaining answer choices, statement II appears in each, so it must be true. Let’s now turn our attention to statement III. Statement III, much like statement I, does not fit with the provided example of a performative utterance, which must be equivalent to “the very execution of the act itself.” Since statement II is the only true statement, choice (B) is the correct answer. Roman numeral I – False: Statement I, a plea for help, does not fit the definition of a performance utterance. Roman numeral II – True: This statement is equivalent to “the very execution of the act itself”—the nature of a performative utterance. Roman numeral III – False: Statement III does not fit with the provided example of a performative utterance. (A) Opposite. Statement I is false. (C) Opposite. Statements I and III are false. (D) Opposite. Statement III is false. 84. (A) A quick passage scan reveals that, according to paragraph 2, “the word [li] in its root meaning is close to ‘holy ritual,’ ‘sacred ceremony.’” The correct answer must accurately reflect these synonyms. Of the given answer choices, the only description that fits with the tone of the paragraph is that which mentions ceremonies as an expression of respect. Indeed, choice (A) is the correct answer. (B) Out of scope. There is no reference in the passage to a divine being which could provide such sanctions. (C) Out of scope. Refined society is not mentioned in the passage. (D) Out of scope. According to the author, li can be performed by anyone, not just holy people.

45

AAMC Test 3 Explanations

85. (A) If it is true that people bind themselves through words and ceremony, then the correct answer that weakens the author’s argument will provide an instance in which people do not bind themselves through words or ceremony. If people do not try to follow up on their moral contractions, then the author’s claim is weakened. Our prediction matches with choice (A), the correct answer. (B) Opposite. If people rarely act ceremoniously, it does not suggest that such rare instances are not, in themselves, binding. In fact, due to such scarce occurrences, it can be inferred that such people are taking their morally binding ceremonies more seriously, and only engaging in them when they know that they can fulfill the binding promise. (C) Out of scope. Whether or not people respond to force has no bearing on the idea that people bind themselves through words and ceremony. (D) Opposite. If people avoid making promises, it does not suggest that such rare instances are not, in themselves, binding. In fact, due to such scarce occurrences, it can be inferred that such people are taking their morally binding ceremonies more seriously, and only engaging in them when they know that they can fulfill the binding promise.

Passage VII (Items 86–92) Passage Map Passage Type: Humanities Topic: Greek religion Scope: How the gods became denigrated Purpose: To illustrate that, while belief in the gods lessened, religion remained vital to early Greece Mapping the Passage: ¶1: Greeks were traditionally religious; over time, gods were discredited; 5th century (Euripides, Socrates) ¶2: This continued into early Hellenistic age (Stoics, sculptors, poets; Callimachus and Theocritus) ¶3: Euhemerus: gods were once human kings; Strato, Menander: gods don’t help with daily life, we don’t need them; St. Paul: Hellenism had no hope or God ¶4: Author: St. Paul’s impression was wrong: gods changed to Divine Saviours. Religion vital in Hellenistic world. 86. (A) Where does the passage discuss Euripides? According to our passage map, in paragraph 1, we read that when Euripides was “not interpreting the gods as profound psychological forces,” he was “presenting them as shady seducers or discredited figures of fun.” Thus, the most favorable portrayal by Euripides is that the gods were psychological forces. Based on this, choice (A) is the correct answer. (B) Faulty use of detail. Euripides did not describe the gods as Divine Saviours. Rather, this reference is made by the author in paragraph 4. (C) Faulty use of detail. The author describes “the inquisitive spirit of Euripides” in paragraph 1. Thus, this is not a way in which Euripides described the gods. (D) Opposite. According to the passage, this is the least favorable portrayal that Euripides gave of the Olympian gods.

46

Verbal Reasoning

87. (D) Where does the passage discuss Socrates and his ideas? Our passage map focuses our attention to paragraph 1, where we learn that Socrates “secured his death sentence [for] ‘not believing in the gods in whom the city believes.’” If Socrates was found to have said that he would obey his god, rather than his Greek contemporaries (who incidentally still worshipped the Olympian gods), then the statement would support the claim presented in the item stem only if it can be shown that Socrates believed in a god other than the city gods. Our prediction matches with choice (D), the correct answer. (A) Opposite. This new finding would support the claim stated in the item stem. (B) Out of scope. The new evidence only conditionally supports the author’s claim—that is, it is only relevant if the god that Socrates is referring to can be shown to be different from the city gods. (C) Opposite. If Socrates’ god were also an Olympian god, Socrates would have been obeying the city gods. If true, then this new information would weaken the claim that Socrates didn’t believe in the city gods. 88. (A) Where are the Stoics mentioned in the passage? According to our passage map, in paragraph 2, we read that the Stoics “reinterpreted and accommodated many individual deities as merely allegorical explanations of natural phenomena.” Thus, if it is found that the Stoics originated the idea of Zeus as the source of thunder, then this would agree with the claim made in the passage. As such, choice (A) is the correct answer. (B) Distortion. In paragraph 2, we read that the Stoics regarded the Greek deities as explanations of natural phenomena, and are contrasted sharply with the many other people who saw the gods as merely symbols. (C) Opposite. The author tells us that, although the Stoics came to see the gods as allegorical explanations, the Stoics still believed in the divine Providence. Thus, this new claim doesn’t weaken that assertion. (D) Faulty use of detail. These “slighting references to the Olympian powers” are attributed to people during a period of time: the early Hellenistic age. The Stoics were only a group that lived during that time. The author does not make any claims showing that Stoics were the ones making such slighting references. 89. (B) Where does the passage make this claim? A brief skim of the passage shows that paragraph 4 concludes by saying that “religion…turned out to be one of the most vital elements in the Hellenistic world.” Why? Although most of the passage addresses the disintegrating faith in traditional Greek gods, paragraph 4 argues that the Greeks “no longer pinned [their] faith on those gods, but on a number of Divine Saviours.” It is during this discussion of Divine Saviours that the author makes the assertion regarding the importance of religion in the Hellenistic world. Therefore, we expect the correct answer to reference these Divine Saviours as reason for the importance of religion. As such, choice (B) is the correct answer. (A) Opposite. St. Paul characterized the Hellenistic world as being “without God,” indicating that religion was not vital. (C) Opposite. In paragraph 3, these individuals discredited the gods, suggesting that religion was not vital. (D) Opposite. This is the claim made by Euhemerus and stands as evidence that the gods were losing their grip on Hellenes. This would indicate that religion was not vital.

47

AAMC Test 3 Explanations

90. (C) A quick passage scan reveals that the Iliad and the Homeric Hymns are mentioned in paragraph 1 as examples that there “had always been a tendency…to treat the gods with a certain familiar flippancy.” Thus, these literary works are presented as evidence so as to substantiate the author’s conclusion. Our prediction matches nicely with choice (C), the correct answer. (A) Faulty use of detail. In paragraph 1, the author tells us that the rationalist movement was “of the later fifth century,” occurring after the two works in question. Therefore, these literary works could not have been used as evidence for this claim. (B) Faulty use of detail. In paragraph 2, the author tells us that “the poets Callimachus and Theocritus showed that…the old gods were no longer a matter of belief or serious concern.” Such a conclusion was related to these poets, not to those involved with the Iliad and the Homeric Hymns. (D) Faulty Use of Detail. This claim is made in paragraph 3, and therefore is not related to the conclusion regarding the Iliad and the Homeric Hymns. 91. (D) Where does the author mention these names? According to our passage map, paragraph 2 says that, “like Hellenistic sculptors, who began to represent some of these gods…in much less idealistic forms than those their predecessors had favored, the poets Callimachus and Theocritus showed that…the old gods were no longer a matter of belief or serious concern.” This implies that, prior to Callimachus and Theocritus, things were more idealized. The only choice that fits with our prediction is choice (D), the correct answer. (A) Out of scope. There is no indication that portraits and sculptures were similar, but rather that they were more idealized. (B) Out of scope. Based on the passage, the only indication of what portraits were like before the Greek poets Callimachus and Theocritus is that they were more ideal. (C) Out of scope. Based on the passage, the only indication of what portraits were like before the Greek poets Callimachus and Theocritus is that they were more ideal. 92. (B) Throughout the passage, the author refers to reasons why the gods became less and less of an influence in Hellenistic life. Specifically, in paragraph 3, the author cites Menander and Epicurus stating that the “traditional gods seemed able to do nothing to ease people’s daily…life.” An Athenian hymn considered them “at least indifferent….” As such, the gods lost their power because they did not seem relevant or helpful to people. Why did the gods become relevant once again as stated in paragraph 4? They did so because their concept gave way to Divine Saviours, which gave strength and holiness during mortal life, and hope for immortality and happiness after death. In order for religion to retain its vitality, then, it must respond to the needs of the people. Based on our prediction, choice (B) is the correct answer. (A) Out of scope. The idea of miraculous gifts isn’t a part of the author’s argument. (C) Faulty use of detail. The rationalists are not part of the argument for how religion was maintained as a vital force; indeed, they illustrate the decline of religion. (D) Out of scope. There is no assertion made that suggests that religion must be attractive to such a defined and distinctive type of people.

48

Biological Sciences

Biological sciences ANSWER KEY 95. C

108. D

121. B

134. C

96. B

109. B

122. A

135. D

97. C

110. A

123. C

136. B

98. D

111. C

124. D

137. C

99. D

112. A

125. C

138. D

100. D

113. B

126. A

139. B

101. D

114. A

127. C

140. A

102. A

115. D

128. B

141. D

103. B

116. B

129. B

142. D

104. B

117. C

130. A

143. A

105. A

118. D

131. A

144. D

106. B

119. B

132. C

145. C

107. B

120. D

133. D

146. B

49

AAMC Test 3 Explanations

AAMC Test 3 biological Sciences—Kaplan Explanations Passage I (Items 95–99) Passage Map Passage Type: Experiment Topic: Cell differentiation Scope: Experiments to characterize the process of cell differentiation ¶1: Two experiments to study process by which fertilized egg differentiates ¶2: Mosaic Hypothesis: cell fate determined by apportioned genes, independent of external stimuli ¶3: Regulative Hypothesis: each cell has potential to become complete organism; fate dependent on external stimuli/position 95. (C) The item requires us to have a good understanding of the Mosaic Hypothesis, so let’s start by reviewing the passage. According to our map, paragraph 2 states that a cell’s fate is determined by apportioned genes— differentiation is independent of external conditions. As such, in the experiment regarding the two-celled frog embryos, when one of the cells of an embryo was killed, only half of an embryo developed from the remaining cell. This suggests that each differentiated cell is given only the information necessary for its own development. Looking at the answer choices, we notice that only choice (C) mirrors our discussion of the Mosaic Hypothesis— when the cells are separated, only a partial embryo forms. Note that the other answer choices support the Regulative Hypothesis—again, choice (C) is the correct response. (A) Opposite. This suggests that many cells receive the information necessary to code for the development of a complete embryo—the Mosaic Hypothesis suggests that such information is only passed onto certain cells on a “need-to-know” basis. (B) Opposite. This suggests that more than one cell receives the information necessary to code for the development of the head—the Mosaic Hypothesis suggests that such information is only passed onto certain cells on a “need-to-know” basis. (D) Opposite. This statement supports the Regulative Hypothesis: any cell has the potential to become any other type of cell. This would indicate that each cell contains identical genetic information—contrary to what the Mosaic theory proposes. 96. (B) This item presents us with a hypothetical experiment and asks us to determine which of the two hypotheses is supported by the new evidence. Characterizing the answer choices, we notice that two of them are in favor of the Regulative Hypothesis, whereas the other two support the Mosaic Hypothesis. We are told that the nucleus of a frog egg is replaced by the nucleus of a differentiated cell. The egg then develops into an adult frog. In order for this to occur, the nucleus of a differentiated cell (the tadpole gut cell) must contain all of the genetic information needed to develop into every type of cell found within the organism—such an idea is supported by the Regulative Hypothesis. Glancing at the answer choices, we notice that choice (B) mirrors our prediction, and is thus the correct response.

50

Biological Sciences

(A) Distortion. Radiation would not allow the frog egg to develop into a complete organism. (C) Opposite. The frog egg was able to develop into an adult frog from the nucleus of a differentiated cell, indicating that the differentiated cell contained all of the genetic information needed to produce an entire organism—this supports the Regulative Hypothesis. Also, there is no reason to believe that the nucleus retained any of its previous genes after radiation. (D) Opposite. The frog egg was able to develop into an adult frog from the nucleus of a differentiated cell, indicating that the differentiated cell contained all of the genetic information needed to produce an entire organism—this supports the Regulative Hypothesis. Such information contradicts the Mosaic Hypothesis, which argues that each differentiated cell is only apportioned the information that is necessary for its own development. 97. (C) What would support the validity of the Regulative Hypothesis? Let’s quickly refer to our map to answer this question. The Regulative Hypothesis, discussed in paragraph 3, states that each cell has the potential to become any of the cells found within the complete organism—its fate is dependent on external stimuli, as well as position in the embryo. We expect, thus, that the correct answer will prove some aspect of this dependency. By showing that a cell’s development was dependent on its position, the evidence presented in choice (C) would best support the Regulative Hypothesis. (A) Opposite. This would support the Mosaic Hypothesis, which states that each differentiated cell is only apportioned the information that is necessary for its own development. (B) Opposite. The Regulative Hypothesis suggests that a cell’s fate is dependent on external stimuli— environmental factors and position in the embryo. (D) Opposite. This would support the Mosaic Hypothesis, which states that each differentiated cell is only apportioned the information that is necessary for its own development. 98. (D) This item asks us to compare the two hypotheses presented in the passage in order to determine which one is most consistent with our current understanding of cell differentiation. What do we know about human differentiation? Human differentiation is based on the idea that developing cells release factors into the surrounding extracellular fluid; these factors signal other cells to develop in a certain manner—a process known as induction, since one cell can induce another cell to develop in a directed manner. As such, the development of human embryonic cells is dependent on environmental factors and positioning within the embryo—this represents the fundamental concepts of the Regulative Hypothesis. From the given answer choices, only choice (D) matches our rationale. (A) Faulty use of detail. Although it is true that germ cells lose half of their genetic material during meiosis, the embryo receives a full complement of genes upon fertilization—this statement is unrelated to either theory. (B) Opposite. Although it is true that specific genes are activated during the differentiation of each cell type, this is not consistent with the Mosaic Hypothesis. The Mosaic Hypothesis argues that each differentiated cell is only apportioned the information that is necessary for its own development. We know, however, that each human cell contains 46 chromosomes, regardless of its differentiation. (C) Distortion. The Regulative Hypothesis states that differentiation is dependent on cell position.

51

AAMC Test 3 Explanations

99. (D) We are presented with a diagram of an embryonic two-cell nucleus and asked to determine which of the four figures represents the cell nucleus at the 64-cell stage. At the two-cell stage, the nucleus is diploid (it contains 2n chromosomes). As the cells divide, they undergo mitosis to create a copy of each chromosome, which will be apportioned equally among the daughter cells. Therefore, each daughter cell should have the same number of chromosomes as the parent cell. Based on this, we predict that a cell at the 64-cell stage of an embryo would contain the exact number of chromosomes as the cell at the two-cell stage—only choice (D), the correct answer, depicts this situation. (A) Distortion. This would be the haploid nucleus of a daughter cell resulting from meiosis, not mitosis. (B) Distortion. This would be the haploid nucleus of a daughter cell resulting from meiosis, not mitosis. (C) Distortion. This nucleus has 4n chromosomes; this is similar to a cell’s appearance after the S phase (DNA replication), which occurs before cell division. However, in this phase, although the amount of DNA is also doubled, the cell still contains the diploid number. 100. (D) This item asks us for experimental evidence in support of the Mosaic Hypothesis, so let’s start by reviewing this hypothesis. According to our map, paragraph 2 states that a cell’s fate is determined by apportioned genes—differentiation is independent of external conditions. As such, in the experiment regarding the twocelled frog embryos, when one of the cells of an embryo was killed, only a partial embryo developed from the remaining cell. This suggests that each differentiated cell is given only the information that is necessary for its own development. We expect the correct answer to further validate some aspect of this theory. If it is found that the fate of transplanted cells is unaffected by their new position in the embryo, then this would strengthen the Mosaic Hypothesis—choice (D) is the correct answer. Note that the other answer choices support the Regulative Hypothesis. (A) Opposite. This would support the Regulative Hypothesis, which states that each cell has the potential to become any of the cells found within the complete organism—its fate is dependent on external stimuli, as well as position in the embryo. (B) Opposite. This would support the Regulative Hypothesis, which suggests that the cells interact to regulate development. (C) Opposite. This would support the Regulative Hypothesis, which states that each cell has the potential to become any of the cells found within the complete organism—its fate is dependent on external stimuli, as well as position in the embryo.

52

Biological Sciences

Passage II (Items 101–106) Passage Map Passage Type: Information Topic: Cholesterol Scope: Familial hypercholesterolemia ¶1: Cholesterol: component of plasma membranes, used to synthesize steroid hormones ¶2: Familial hypercholesterolemia—disease may be moderate or severe ¶3: No difference in dietary habits between affected and unaffected individuals ¶4: HC caused by lack of membrane protein 101. (D) This item tests our understanding of hormone synthesis. According to our map, the first paragraph states that cholesterol is a precursor to steroid hormones—the stem is really just asking us to identify the steroid hormone among the various answer choices. The classic steroid hormones whose precursors are cholesterol are estrogen, progesterone, and testosterone. From the given answer choices, only estrogen is mentioned, making choice (D) the correct response. (A) Opposite. Insulin is a peptide hormone. (B) Opposite. Gastrin is a peptide hormone. (C) Opposite. Thyroxin is a peptide hormone. 102. (A) Glancing at the answer choices, we understand that the question is asking us whether HC has a genetic component to it. Generally, we can recognize a genetic disorder by the fact that it runs in families and is typically independent of environmental factors. What information does the passage provide regarding HC? According to our map, the third paragraph states that HC is common in some families and absent from others. It also states that there is no difference among the dietary habits of affected and unaffected families. Based on these observations, we can predict that HC is caused by a genetic disorder—it cannot be linked to any environmental influences. From the given answer choices, only choice (A) matches our prediction. (B) Opposite. Paragraph 3 suggests that there is “no significant difference between the dietary habits of individuals in affected and unaffected families.” (C) Opposite. Paragraph 3 suggests that there is “no significant difference between the dietary habits of individuals in affected and unaffected families.” (D) Opposite. A defective protein is typically caused by a defect in its genetic coding—its DNA sequence. 103. (B) How can we mimic the effects of HC in a healthy individual? To answer this question, we must first understand what causes HC. According to our map, the last paragraph of the passage discusses the differences between healthy and affected individuals. Here, we are told that individuals with HC lack a membrane protein that normally binds to LDL. Therefore, in order to reproduce this condition, the researcher must mimic the absence of this

53

AAMC Test 3 Explanations

membrane protein. This could be accomplished by administering a drug that binds to the membrane protein, impeding its ability to bind LDL. We predict, therefore, that providing a drug that attaches to the plasma membrane protein that normally binds LDL will prevent the healthy person’s LDL from binding the protein, thereby imitating the disease. The answer choice that matches our prediction is choice (B), the correct answer. (A) Distortion. There is no information in the passage that suggests that the absorption of lipids from the small intestine is affected in HC. (C) Distortion. There is no information in the passage that suggests that the production and secretion of bile is affected in HC. (D) Distortion. Not all plasma membrane proteins are altered in HC—only those that bind LDL. 104. (B) The item describes the cholesterol levels of the parents and their child and asks us to determine the nature of this disease—the answer choices suggest that HC could be the result of a dominant, codominant, or recessive allele. According to our map, the second paragraph tells us that a cholesterol level of 3.0 mg/mL is typical of moderately affected individuals, whereas a level of 7.0 mg/mL describes those that are severely affected. Glancing back at the item stem, we can conclude that, whereas the parents are moderately affected, their child has a severe form of HC. What does this mean about the dominance behavior of HC? If the HC allele was simply dominant to the normal allele, the parents who have clearly passed the HC gene onto their offspring would also be expected to show similarly high levels of cholesterol. However, since the parents are only moderately affected, the HC gene cannot be considered completely dominant—we can eliminate choices (C) and (D). Due to its behavior, we’ll use “C” to denote the HC gene and “c” to denote the normal gene. Using this designation, the parents and their child have the respective genotypes: Cc and CC. Since Cc results in a moderately affected individual, whereas CC causes a severe form of HC, we conclude that the HC allele is codominant. Had HC been due to a recessive allele, the parents would have shown no signs of the disease (normal cholesterol levels), while their child would have been severely affected. We conclude that HC is due to a codominant allele, making choice (B) the correct answer. (A) Distortion. The additive nature of the disease suggests that multiple copies of the disease-allele contribute to a more severe form of HC than seen in those with just one allele. As such, the disease must be codominant, not recessive. Additionally, if the disease was the result of a recessive allele, then no symptoms would be observed in the parents. (C) Distortion. The additive nature of the disease suggests that multiple copies of the disease-allele contribute to a more severe form of HC than seen in those with just one allele. As such, the disease must be codominant, not completely dominant. (D) Distortion. The additive nature of the disease suggests that multiple copies of the disease-allele contribute to a more severe form of HC than seen in those with just one allele. As such, the disease must be codominant, not completely dominant. 105. (A) The item asks us how to reduce fat absorption from the small intestine into the bloodstream—this is a pseudodiscrete question, since we do not need any information from the passage to answer it. For this question, we must understand how fat is absorbed. Small fatty acids are absorbed directly by the intestinal capillaries and enter the portal circulation. Large fatty acids, glycerol, and cholesterol diffuse into the mucosal cells of the small intestine. Once inside, the fatty acids and glycerol recombine to form triglycerides. The triglycerides and cholesterol are packaged into chylomicrons, which are then secreted into lacteals, small lymphatic vessels within

54

Biological Sciences

the intestinal villi. The lacteals transport the fats through the lymphatic system—which converges with venous blood at the thoracic duct. Therefore, the most effective way to prevent fat absorption from the small intestine into the bloodstream is to either prevent movement into the mucosal cells, the formation of chylomicrons, or the absorption of chylomicrons by lacteals. From the given answer choices, only choice (A), vasoconstriction of the lacteals inside intestinal villi, matches our prediction. (B) Distortion. The capillaries in the smooth muscle provide blood to the small intestine so as to provide energy for peristalsis. Constricting these vessels will only slow peristalsis, not prevent absorption of fats. (C) Distortion. The peritoneum is not in contact with the lumen of the small intestine and is therefore not involved in absorption. (D) Distortion. The peritoneum is not in contact with the lumen of the small intestine and is therefore not involved in absorption. 106. (B) How can we prove that a disease has a genetic basis? In order for a disease to be considered genetic, it cannot be caused by environmental and behavioral factors. The best way to show that a disease is not caused by environmental and behavioral factors is to compare the environment and behavioral habits of affected and unaffected individuals. If the environment and behavioral habits are similar, then the disease can be attributed to genetic factors. What does the passage tell us about the causes of HC? According to our map, paragraph 3 states that there is no significant difference between the eating habits of affected and unaffected individuals. In other words, environmental and behavioral factors alone cannot determine if a person will get HC or not, which means that HC must be genetically transmitted. The answer choice that matches our prediction is choice (B), the correct response. (A) Distortion. This statement alone does not exclude dietary habits as a cause of the disease. According to this statement, HC could be due to heredity, or it could be attributed to the similar activities/ food consumed by the families, since they live in such close proximity. Again, this statement does not rule out environmental factors. (C) Distortion. The malfunction of a protein in the plasma membrane could be caused by toxins or other environmental factors. (D) Distortion. The absence of cholesterol-containing endocytotic vesicles could be caused by toxins or other environmental factors.

Discrete Set I (Items 107–109) 107. (B) Based on the item stem, we know that we are being tested on our knowledge of Mendelian genetics. The answer choices are numerical in nature, indicating that our task is to calculate how many short plants would be expected in the offspring generation. We are told that the short gene is recessive and the tall gene is dominant. The item stem also mentions that after the first breed, 787 plants had a tall phenotype. How can we calculate the number of short plants? In a cross where both parents are heterozygous, some of the offspring will express the recessive trait—we can eliminate choice (A). We also expect that the offspring with the recessive trait will be much fewer in number than those with the dominant phenotype—we can eliminate choices (C) and (D), leaving choice (B) as the correct answer. Let’s confirm our initial prediction with the use of a Punnett square. If we use the designation “T” for tall and “t” for short, the heterozygous parents would be crossed accordingly:

55

AAMC Test 3 Explanations

T

t

T

TT

Tt

t

Tt

tt

Of the resulting offspring, _​ 34 ​ will be tall (TT, Tt) while the remaining ​ _14 ​ will be short. The number 787 actually represents ​ _34 ​ of the next generation, or approximately 1,000 offspring. Out of these 1,000 plants, approximately 250 would be expected to be short—again, choice (B) is the correct answer. (A) Miscalculation. In a heterozygous cross, some of the offspring will express the recessive trait. (C) Miscalculation. The number of offspring with the short phenotype has to be fewer than those with the tall phenotype. (D) Miscalculation. This number is more than the total population of offspring. 108. (D) In spite of the complicated nature of the item stem, we are essentially asked to determine the location of glucose in the cell during its breakdown into carbon dioxide, water, and ATP. The answer choices make reference to the cytoplasm, mitochondria, and the nucleus. Let’s quickly review the path that glucose takes as it is metabolized. The first step in the breakdown of glucose is glycolysis, which occurs in the cytoplasm—already, choice (D) must be the correct answer. Let’s follow this process further to validate our answer choice. The product of glycolysis is pyruvate, which is transported into the mitochondrial matrix where it is decarboxylated, generating acetyl CoA. This molecule then enters the citric acid cycle. We would expect, therefore, for the 14C label to be found first in the cytoplasm, and then the mitochondria—choice (D) is the correct response. (A) Distortion. Glycolysis occurs in the cytoplasm. (B) Distortion. Glucose does not enter the nucleus during its metabolism. (C) Distortion. The ribosomes are not involved in the metabolism of glucose. 109. (B) The item asks us why an undernourished individual would be excreting extensive amounts of nitrogen in the urine. We are told that this person was only able to eat starches, which does not satisfy the body’s metabolic requirements. People who are undernourished enter a catabolic state, breaking down their own macromolecules so as to provide glucose to the brain. Which macromolecules contain nitrogen? Proteins are the main source— they are metabolized preferentially in the absence of sufficient carbohydrates and fats. As such, this individual has been breaking down the proteins of his own body in order to provide glucose to the brain. The only answer choice that correctly depicts this scenario is choice (B), the correct answer. (A) Distortion. Glycogen stores typically last only 12–24 hours. Also, glycogen does not contain nitrogen. (C) Distortion. Fat does not contain nitrogen. (D) Distortion. One function of the kidneys is to excrete nitrogenous wastes. Excretion of a large amount of nitrogen does not indicate kidney failure but rather a high production of nitrogen—as a consequence of the body breaking down its own proteins as a source of glucose.

56

Biological Sciences

Passage III (Items 110–114) Passage Map Passage Type: Information Topic: Carvone Scope: Separation of carvone and limonene ¶1: Carvone enantiomers have different characteristic odors. ¶2: Chemist tries to separate (+)-carvone and limonene. ¶3: (+)-carvone and limonene were separated by vacuum fractional distillation; very different boiling points Fig. 1: Schematic of vacuum fractional distillation apparatus 110. (A) The item asks us to determine which compound distills, or evaporates, first when heated. The compound with the lower boiling point will distill first, so we must determine which compound has the lower boiling point. However, simply knowing this information allows us to eliminate choices (B) and (D), since they incorrectly state that the first compound to distill will have a higher boiling point. Analyzing the structures of carvone and limonene, we notice that they are both cyclohexenes, but that carvone has a carbonyl group. The polarity of the carbonyl group creates a dipole moment in the molecule, increasing the strength of the intermolecular forces that exist in solution, and causing a rise in the boiling point. Because of this, carvone will have a higher boiling point than limonene—we expect limonene to distill first, making choice (A) the correct answer. (B) Distortion. Limonene distills first because of its lower boiling point. (C) Opposite. Carvone has the higher boiling point, and will distill last. (D) Opposite. Carvone has the higher boiling point, and will distill last. 111. (C) The item asks us to determine the purpose of the ebulliator. Since this is a detail item, let’s refer to our passage map to find the specific location where the ebulliator is discussed. Our passage map informs us that the apparatus is described in paragraph 3. Here, we read that the ebulliator is used to introduce small air bubbles into the system. The air bubbles break the surface tension of the liquid. Why is this important? This prevents superheating by lowering the surface tension of the liquid, allowing the liquid to evaporate. Superheating is the phenomenon of heating a liquid past its boiling point without causing evaporation, which can result in bumping, or a sudden burst of vapor. Superheating and bumping both decrease the quality of separation. Thus, the function of the ebulliator is similar to that of a boiling chip used in chemistry experiments; to prevent the liquid from superheating and bumping. The answer choice that mirrors our line of reasoning is choice (C), the correct answer. (A) Distortion. The ebulliator is located in the distillation flask, not the receiving flask. (B) Distortion. The vacuum is created by the vacuum source, not the air bubbles introduced into the system. (D) Distortion. The vacuum source keeps the pressure inside the system low.

57

AAMC Test 3 Explanations

112. (A) The item asks us to identify a way to improve the separation between the two compounds. From the passage, we know that limonene and (+)-carvone are separated using fractional distillation. Fractional distillation is generally used for separating liquids that boil less than 25°C apart. The column is filled with inert objects, like glass beads, on which the vapors condense, which then re-evaporate and condense further up the column. The series of condensations and evaporations successively separates the liquids. Each time the liquid evaporates it contains a greater proportion of the lower-boiling point liquid. When the vapor is composed of only one component, it will condense in the distillation column and be collected in the receiving flask. Several modifications can make the separation better. First, the fractionating column can be longer, so the liquid condenses and re-evaporates multiple times. Additionally, a higher pressure will raise the boiling points of both liquids, increasing the difference between the two boiling points. Also, heating the flask at a slower rate will decrease the rate at which the liquid boils and increase the amount of time spent in the fractionating column. Of the choices given, only choice (A) matches with one of the mentioned improvements, and is thus the correct response. (B) Opposite. A lower pressure will decrease the boiling point of each liquid and narrow the difference between the boiling points, resulting in worse separation. (C) Distortion. The condenser is where the vapor condenses after it has been separated from the other liquid. Anything done at this point will have no effect on separation. (D) Opposite. A shorter fractionating column would limit the number of vaporizations and condensations, resulting in worse separation. 113. (B) The item stem illustrates a 2-D structure of carvone and asks us to determine which carbon’s orientation (carbon 2, 5, or 7) is responsible for creating two enantiomers. To answer this question, we must fist recognize that (+)carvone and (–)-carvone are enantiomers—they differ in orientation around a chiral carbon. A chiral carbon is a carbon bound to four different substituents—for a carbon to be chiral, it must have only single bonds. Thus, our task is to identify which of the three listed carbons is chiral. Glancing at the structure of carvone, we notice that only C-5 is a chiral carbon—carbons 2 and 7 both participate in double bonds, so they are not chiral. Therefore, only carbon 5 can have a different 3-D arrangement, resulting in the different enantiomers of carvone—the correct answer is illustrated in choice (B). (A) Distortion. Carbon 2 is not chiral because it participates in a double bond. (C) Distortion. Carbon 2 is not chiral because it participates in a double bond. (D) Distortion. Carbons 2 and 7 are not chiral because they each participate in a double bond. 114. (A) The item requires us to ascertain what will happen to the boiling points of the liquids if a leak develops in the vacuum distillation system. The answer choices offer qualitative comparisons for the boiling points of the two compounds. Let’s reason through this question using our knowledge of boiling point and vacuum distillation. If a leak develops in the vacuum distillation system, the pressure within the system will increase. As such, the question is really asking what happens to the boiling point of a substance in increased atmospheric pressure. Knowing that the boiling point of a substance depends on the external pressure and that all compounds are affected by a change in pressure allows us to eliminate choices (C) and (D). How does the boiling point of a substance depend on the ambient pressure? The boiling point of a liquid is the point where the vapor pressure of the liquid is equal to the atmospheric pressure. If the atmospheric (or surface) pressure increases, then the vapor pressure must increase

58

Biological Sciences

further before equating the ambient pressure—to increase the vapor pressure, the temperature must be increased. As such, the boiling point increases as atmospheric pressure increases. With regards to our compounds, their boiling points will increase, though at different rates. As their boiling points increase, the difference between the boiling points will become more exaggerated—choice (A) is the correct response. (B) Opposite. Because the atmospheric pressure increases, an increased temperature is required to increase the vapor pressure of the liquid. (C) Distortion. Because the atmospheric pressure increases, an increased temperature is required to increase the vapor pressure of the liquid. (D) Distortion. The boiling points of both components will increase, though at different rates. As the boiling points increase, the difference between the boiling points will become more exaggerated.

Passage IV (Items 115–119) Passage Map Passage Type: Experiment Topic: Ca2+ Scope: Control of Ca2+ entry into the cell by Hormone X ¶1: Hormone X causes cells to absorb more Ca2+; some males have a defect in their ability to respond to Hormone X ¶2: Series of experiments to determine hormone pathway and point of defect in males List: Results from experiments 115. (D) From the wording of the item stem, we know that this is a genetics question. We are told that Hormone X is X-linked and asked to determine what proportion of the children of an affected male would inherit the father’s defective gene. To solve this problem, we need to know if Hormone X is dominant or recessive. In paragraph 1, we are told that women in the affected family had no defects in their ability to respond to Hormone X. As such, the defect is likely X-linked recessive. The question only tells us that the father is affected, so we can assume that the mother is normal (homozygous). Next, we’ll create a Punnett square for an X-linked recessive gene with an affected male (XhY) and unaffected female (XX): X

X

Xh

XXh

XXh

Y

XY

XY

Analyzing the results, we notice that none of the children will be affected. Since all of the males get their X chromosome from their mother, the affected allele cannot be transmitted from father to son. Similarly, all females get the affected allele from their father, and a normal X chromosome from their mother, so females will be carriers but not affected. We predict that none of the children will be affected—choice (D) is the correct answer. (A) Distortion. All males get their X chromosome from their mother, so the allele is not transmitted from father to son. All females get a normal X chromosome from their mother, so females will be carriers but not affected.

59

AAMC Test 3 Explanations

(B) Distortion. All males get their X chromosome from their mother, so the allele is not transmitted from father to son. All females get a normal X chromosome from their mother, so females will be carriers but not affected. (C) Distortion. All females get a normal X chromosome from their mother, so females will be carriers but not affected. 116. (B) The item asks us to determine which protein utilizes ATP for its action. Glancing at the answer choices, we notice that we have to choose between Hormone X, Protein R, and Protein P. A protein that phosphorylates another molecule is likely to use ATP as its phosphate source. Where in the passage is the function of each protein discussed? According to our map, the experimental results numbered in the passage as 1–6 contain the data that explains the role of each of the proteins listed in the answer choices. Result #4 tells us that protein R phosphorylates protein P. As such, Protein R must be using ATP for its action, allowing us to match our prediction with choice (B), the correct response. (A) Distortion. According to result #2, the function of Hormone X is to bind Protein R; this is not likely to require ATP. (C) Distortion. Protein P is phosphorylated by Protein R, indicating that Protein R, and not Protein P, requires ATP for its action. (D) Distortion. According to result #2, the function of Hormone X is to bind Protein R; this is not likely to require ATP. In addition, Protein P is phosphorylated by Protein R, indicating that Protein R, and not Protein P, requires ATP for its action. 117. (C) The item asks us to determine the function of phosphorylated Protein P. Where in the passage can we find this information? Once more, the experimental results numbered 1–6 contain the information that we need. Specifically, result #6 tells us that “phosphorylated Protein P was necessary and sufficient to increase the rate of Ca2+ entry in cells.” This means that the only thing needed to let Ca2+ into the cell is phosphorylated Protein P. Of the given choices, only a membrane channel will allow Ca2+ into the cell—the correct answer is choice (C). (A) Distortion. Although a sodium/potassium pump allows sodium and potassium to enter and leave the cell, it will not affect intracellular calcium levels. (B) Distortion. A phosphatase cleaves a phosphate group from a molecule; it may or may not affect intracellular calcium levels—there is insufficient information provided to make such a conclusion. (D) Distortion. Protein R is a hormone receptor, not Protein P, since it binds Hormone X (result #2). 118. (D) The item requires us to have a good understanding of the mechanism of action of Hormone X. How can we stop the influx of Ca2+? Let’s review the results in the passage to better understand what factors influence the entry of calcium. According to result #6, phosphorylated Protein P is necessary for calcium entry into the cell. Conversely, if Protein P was to lose its phosphate group, calcium entry would cease. We predict that dephosphorylation of Protein P will prevent the influx of Ca2+—choice (D) is the correct answer. (A) Distortion. If Hormone X no longer binds to Protein R, then further phosphorylation of Protein P molecules would not occur; but the already phosphorylated Protein P molecules would still function, allowing the influx of Ca2+.

60

Biological Sciences

(B) Distortion. If the complex of Hormone X and Protein R was endocytosed (and presumably destroyed), further phosphorylation of Protein P molecules would not occur, but the already phosphorylated Protein P molecules would still function, allowing the influx of Ca2+. (C) Distortion. Without substrate, further phosphorylation of Protein P molecules would not occur; but the already phosphorylated Protein P molecules would still function, allowing the influx of Ca2+. 119. (B) The item requires us to have a good understanding of the results presented in the passage. The answer choices mention replacement of single amino acids or introduction of premature stop codons in either Protein P, Protein R, or Hormone X. Our task is to determine the effect of a mutation in the DNA of Family 2. According to result #5, Protein R could not phosphorylate Protein P isolated from affected males of Family 2. The Protein R used in the experiment was isolated from females, so the Protein R must be functional (because females are unaffected). Therefore, there must be a mutation in Protein P that prevents it from being phosphorylated. Based on this prediction, choice (B) must be the correct response. Indeed, a mutation that prevents protein P from being phosphorylated could originate from the replacement of a single amino acid. Again, the correct answer is choice (B). (A) Distortion. The Protein R used in the experiment was isolated from females, so the Protein R must be functional (because females are unaffected). (C) Distortion. The Protein R used in the experiment was isolated from females, so the Protein R must be functional (because females are unaffected). (D) Distortion. Result #2 states that Hormone X bound to Protein R of all cells examined; Hormone X is normal in the males from Family 2.

Discrete Set II (Items 120–124) 120. (D) This item tests our knowledge of osmoregulation. We are told that an intravenous infusion causes an increase in the concentration of albumin in the blood (serum)—we are asked to determine the consequences. We are told that albumin is the major osmoregulatory protein in the blood. What does this mean? This indicates that it controls the osmotic pressure of the blood. By increasing the amount of albumin in the blood, we have made the blood more concentrated—hypertonic with respect to the fluid outside of the blood vessels. As such, according to the homeostatic response of osmosis, the fluid found in the tissues (mostly water) will cross into the blood vessels so as to alleviate this imbalance in concentration. This happens because albumin is unable to cross the barrier of the vessel. As such, this surge of tissue fluid will decrease the osmotic pressure of the blood. The answer choice that correctly depicts this situation is choice (D). (A) Distortion. Albumin is a protein normally found in the bloodstream; it would not stimulate an immune response. (B) Distortion. Albumin is a protein that cannot cross through the blood vessel to alter tissue albumin levels— instead, water flows into the blood vessel to remedy the concentration inequality. (C) Opposite. The increased osmotic pressure of the blood draws fluid from the tissue into the bloodstream, not the opposite.

61

AAMC Test 3 Explanations

121. (B) This is a straightforward item that tests our knowledge of taxonomy—how organisms are classified. The answer choices offer four different taxonomic classifications, and our task is to determine which of these is the most specific, that is, which contains the smallest number of organisms. The taxonomy system can be remembered by the mnemonic “King Philip Could Order Fairly Good Soup” or “Kings Play Chess On Fine Grained Sand” for kingdom, phylum, class, order, family, genus, and species. Organisms of the same species are very closely related, whereas those belonging to the same kingdom are very distantly related. That is, the further up the taxonomy chain, the further apart the organisms are. Of the choices listed, animals of the same genus would be the most closely related, making choice (B) the correct response. (A) Distortion. Animals of the same genus would be the most closely related. (C) Distortion. Animals of the same genus would be the most closely related. (D) Distortion. Animals of the same genus would be the most closely related. 122. (A) The item stem describes ducklings following any large moving object they encounter during a critical period after hatching. The answer choices all refer to different neuronal mechanisms that are integral to learning and perception. The description in the item stem is the definition of imprinting, a mechanism in which newly hatched birds form social attachments to objects or other animals that they see right after they are born—the correct answer is choice (A). (B) Distortion. Habituation is a non-associative learning in which response to a stimulus decreases over time. (C) Distortion. Conditioning is an associative learning in which a known stimulus is paired with a neutral stimulus; after a period of time the neutral stimulus is able to elicit the same response as the initial known stimulus. The classic example of this is Pavlov training his dog to salivate when a bell is rung—a conditioned response that resulted from ringing the bell every time the dog was fed. (D) Distortion. Discrimination is a perceptual process, not a learning process. 123. (C) We are given four cyclic alkanes and asked to determine which one of them has the lowest heat of combustion, so let’s start our discussion by reviewing what this concept means. The heat of combustion is the energy released when a compound undergoes combustion (a reaction with oxygen which results in the formation of water and carbon dioxide). Thus, the heat of combustion is determined by the energy contained in each bond. We would predict that the heat of combustion will be high when the bond energy is high. In cycloalkanes, part of the bond energy is determined by analyzing the ring strain. Ring strain is determined by angle strain, torsional strain, and nonbonded strain. Because the carbons in cycloalkanes are sp3 hybridized, they would ideally have angles of 109.5° between each bond. Thus, the more the angle is stretched or compressed from this ideal angle, the more strain the compound will have. A higher strain implies higher bond energy and thus higher heat of combustion. With this prediction in mind, let’s take a look at the given choices. Since cyclohexane is able to eliminate all three types of strain in its chair conformation, it is the most stable molecule found in the answer choices, and so has the lowest heat of combustion—the correct answer is choice (C). (A) Distortion. Cyclopropane has angle strain and torsional strain, indicating high-energy bonds – this corresponds to a relatively higher heat of combustion.

62

Biological Sciences

(B) Distortion. Cyclobutane has angle strain, torsional strain, and nonbonded strain—this corresponds to a relatively higher heat of combustion. (D) Distortion. Cycloheptane has torsional strain—this corresponds to a relatively higher heat of combustion. 124. (D) This item tests our understanding of nucleophiles. Glancing at the answer choices, we notice that our first task is to determine whether or not the compound is a good nucleophile, and secondly, to provide an explanation of why this is true. Let’s start by defining nucleophiles. Nucleophiles are electron-rich compounds that are attracted to positively polarized atoms (nucleo- stands for nucleus, that is, positive charges, and -phile means loving, so nucleophiles are compounds that want positive charges). Nucleophiles are frequently strong bases. Generally, nucleophiles attack atoms that have a partial positive charge. In the compound drawn in the item stem, the nitrogen atoms could be nucleophiles because they are electron dense. However, both of the nitrogen-containing groups are tertiary amines; there are three substituents attached to each nitrogen atom. This means that the electron dense nitrogen atoms cannot attack a positively polarized atom because of steric hindrance. We would predict, therefore, that the compound cannot act as a nucleophile because of steric hindrance—choice (D) is the correct answer. (A) Distortion. Although the nitrogen atoms are basic and electron rich, they cannot attack a positively polarized atom due to steric hindrance. (B) Distortion. Although the nitrogen atoms are basic and electron rich, they cannot attack a positively polarized atom due to steric hindrance. (C) Distortion. The aromatic portion of the molecule contributes to the electron density of the nitrogen atoms.

Passage V (Items 125–130) Passage Map Passage Type: Experiment Topic: Plasma clearance Scope: Experiments to study plasma clearance of a substance ¶1: Plasma clearance is determined by comparing the concentration of a substance in plasma and urine ¶2: Tm is the maximum rate at which a substance can be reabsorbed by the kidney. ¶3: Experiments to study the plasma clearance of a Substance A ¶4: Experiment 1 – concentrations of Substance A in plasma and urine were measured as Substance A was infused Fig. 1: Plasma clearance of Substance A and glucose according to plasma concentration ¶5: Experiment 2 – effect of ADH on plasma clearance of Substance A 125. (C) The item asks us for the definition of Tm, the tubular transport maximum. According to our map, Tm is defined in paragraph 2. Here, we are told that Tm is the maximum rate of transport at which a substance can be reabsorbed by the kidneys. Beyond this rate, the substance will appear in the urine. Thus, Tm can be thought of as the

63

AAMC Test 3 Explanations

maximum ability of the kidneys to reabsorb a certain substance—the answer choice that matches with this prediction is choice (C). (A) Distortion. This is the glomerular filtration rate, or GFR, or a substance. (B) Distortion. This is the plasma clearance. (D) Distortion. Tubular transport maximum (Tm) is related to the kidneys, not bladder. 126. (A) We are told that the amount of glucose in the tubules is 125 mg/min and asked to determine exactly how much is excreted in the urine. Where in the passage can we find the information that we need to answer this question? According to our map, paragraph 2 tells us that the Tm of glucose is 320 mg/min. What does this passage value indicate? This is the point at which no more glucose can be reabsorbed and glucose is excreted in the urine. Below this threshold value, all of the glucose is reabsorbed. Thus, at 125 mg/min, we would expect all of the glucose to be actively reabsorbed—there won’t be any glucose in the urine. The answer choice that matches with our prediction is choice (A), the correct answer. (B) Distortion. Below 320 mg/min, all of the glucose is actively reabsorbed—there won’t be any glucose in the urine. (C) Distortion. Below 320 mg/min, all of the glucose is actively reabsorbed—there won’t be any glucose in the urine. (D) Distortion. Below 320 mg/min, all of the glucose is actively reabsorbed—there won’t be any glucose in the urine. 127. (C) The item asks us to assess the effect of a low blood pressure on plasma clearance. The answer choices are long explanations that argue for either an increase or decrease in the plasma clearance of a substance. Let’s start by first examining plasma clearance. Plasma clearance, according to paragraph 1, is the capacity of the kidney to remove a substance from the plasma and excrete it into the urine. So how does blood pressure relate to the kidney and plasma clearance? When blood pressure decreases, the body activates several different mechanisms so as to counteract the change, returning the blood pressure back to normal (homeostasis). Several fluid-retaining hormones are secreted (one of which is ADH) which increase the reabsorption of water and decrease the urine output. When reabsorption of water increases, the reabsorption of multiple other substances also increases, including salts and sugars. As such, we would expect that the decreased urine output would allow for greater reabsorption of Substance A by the kidney. Such increased reabsorption means less excretion of Substance A— lower plasma clearance. With this prediction in mind, we notice that only choice (C) mirrors our line of reasoning, and is thus the correct response. (A) Opposite. Decreased urine output allows for more reabsorption of Substance A by the kidney, decreasing plasma clearance. (B) Distortion. ADH levels will be higher, but it is the decreased rate flow of urine that gives the kidney tubules more time to reabsorb Substance A, not the hormone itself. (D) Faulty use of detail. ADH levels will be higher, but it is the decreased rate flow of urine that gives the kidney tubules more time to reabsorb Substance A, not the hormone itself.

64

Biological Sciences

128. (B) The item prompts us to analyze Figure 1 so as to determine which substance is being cleared from the plasma more rapidly at a concentration of 8 mg/mL. Figure 1 depicts the concentration in the urine versus the concentration in the plasma for Substance A and glucose. Using Figure 1, at a concentration of 8 mg/mL, there are approximately 2 mg/mL of Substance A in the urine and no glucose. As such, Substance A is being cleared from the plasma at this point, whereas glucose isn’t. This means that Substance A is being cleared from the plasma more rapidly. Based on this, we can eliminate choice (C). Our next task is to find an explanation for why Substance A is cleared more rapidly than glucose. Based on Figure 1, we determine that Substance A begins to appear in the urine at a concentration just over 6 mg/mL. In paragraph 2, we are told that Tm is the maximum rate of transport at which a substance can be reabsorbed by the kidneys. Beyond this rate, the substance will appear in the urine. Therefore, the Tm for Substance A occurs at a plasma concentration just over 6 mg/mL. On the other hand, glucose does not appear in the urine until about 10 mg/mL. This indicates that Substance A has a lower Tm and is therefore cleared more rapidly. From the given answer choices, only choice (B) matches our prediction, and is thus the correct response. (A) Distortion. The slope of the line does not affect plasma clearance. (C) Opposite. It is Substance A that has a lower Tm—it is cleared more rapidly, not glucose. (D) Distortion. The slope of the line does not affect plasma clearance. 129. (B) The item prompts us to analyze Figure 1 so as to determine at what plasma concentration glucose reaches its Tm. Paragraph 2 tells us that at the Tm, the substance begins to appear in the urine because it can no longer be completely reabsorbed. According to Figure 1, glucose begins to appear in the urine when it reaches a plasma concentration of 10.0 mg/mL. This is the plasma concentration at which the Tm for glucose is reached. The answer choice that correctly illustrates this rationale is choice (B), the correct response. (A) Distortion. This is the plasma concentration at which Substance A reaches its Tm. (C) Distortion. This is the plasma concentration at which the concentration of glucose in the urine will be 3 mg/mL. (D) Distortion. This is the plasma concentration at which the concentration of glucose in the urine will be 5 mg/mL. 130. (A) The item essentially asks us to determine how high blood pressure affects the excretion of a substance. The item stem prompts us to analyze Experiment 2, but our knowledge of kidney filtration is sufficient to make a prediction. We know that high blood pressure forces more filtrate into Bowman’s capsule, thus increasing the glomerular filtration rate. Since more filtrate is present, more Substance A is present in the filtrate. As a result, the Tm is more likely to be exceeded; Substance A is consequently cleared more readily. We predict, thus, that the higher blood pressure affects the output of Substance A through an increased glomerular filtration rate—the answer choice that mirrors our prediction is choice (A). (B) Distortion. Tm is an intrinsic property of the tubules and is independent of blood pressure. (C) Distortion. Although the increased ADH caused the increased blood pressure by reabsorbing more water, the item is asking about the effect of the high blood pressure, not of the ADH. (D) Distortion. Although the increased ADH caused the increased blood pressure by reabsorbing more water, the item is asking about the effect of the high blood pressure, not of the ADH.

65

AAMC Test 3 Explanations

Passage VI (Items 131–135) Passage Map Passage Type: Information Topic: Osteoporosis Scope: Mechanism and treatment of osteoporosis ¶1: Osteoporosis is a decrease in bone mass; rate of bone mass loss increases in the immediate postmenopausal period ¶2: Bone loss related to parathyroid hormone and calcitonin abnormalities, decreased active vitamin D, and estrogen deficiency (in postmenopausal women) ¶3: Hormone replacement and dietary calcium may prevent osteoporosis in women. 131. (A) This item can be considered a pseudo discrete, since it does not require us to use any specific information from the passage. We are asked to determine why women suffer from osteoporosis more than men. Glancing at the answer choices, only one provides a physical difference between men and women—that of bone density. The other three answer choices mention aspects of men and women that are identical. Indeed, women have a lower bone density. As a result, the natural bone mass loss of 0.5 to 1.0% per year affects women more than men—choice (A) is the correct answer. (B) Distortion. Women and men have the same number of vertebrae. (C) Distortion. Women and men have the same digestive system and renal system, and so benefit from the same mechanisms for calcium uptake. (D) Distortion. Women and men have the same renal system and therefore the same mechanisms of vitamin D production. 132. (C) The item asks us to determine the side effects of an estrogen and progesterone therapy. According to paragraph 3, such therapy helps in the prevention of osteoporosis in women—it consists of estrogen and low progesterone doses. What do these hormones do? In premenopausal women, the ovaries secrete progesterone and estrogen during the menstrual cycle. Estrogen stimulates the endometrial tissue to proliferate, whereas a decrease in progesterone causes endometrial tissue breakdown, resulting in menses. If a postmenopausal woman is taking estrogen and progesterone, then menstruation could resume to some extent—choice (C) is the correct answer. (A) Opposite. Supplementation of estrogen and progesterone prevents breast tissue atrophy. (B) Opposite. Supplementation of estrogen and progesterone prevents vaginal dryness. (D) Distortion. High levels of estrogen suppress lactation. 133. (D) This is a straightforward question that asks us to determine which of the given hormones will be inhibited by calcium—it is a pseudo discrete, since it does not require us to use any specific information from the passage. From our knowledge of the endocrine system, we know that parathyroid hormone stimulates bone breakdown

66

Biological Sciences

and the release of calcium in response to low calcium levels. Thus, this hormone should be inhibited by elevated calcium levels in the blood, so as to prevent an even greater liberation of calcium into the blood—the correct answer is choice (D). (A) Distortion. Growth hormone is unaffected by calcium levels. (B) Opposite. Calcitonin is stimulated by high calcium levels. Calcitonin stimulates osteoblasts to build bone, thus decreasing the calcium level in blood. (C) Distortion. Thyroid hormone is unaffected by calcium levels. 134. (C) The item asks us to determine the effect of estrogen supplementation in a man in relation to his chance of developing osteoporosis. According to our map, the third paragraph suggests that estrogen therapy prevents osteoporosis in postmenopausal women. However, the first paragraph tells us that bone loss is briefly accelerated in postmenopausal women—only later does bone loss return to its previous baseline. This would indicate that bone loss is related to estrogen withdrawal, not simply the presence or absence of estrogen. If bone loss was related only to the absence of estrogen, then men would always exhibit a high rate of bone loss. Since only estrogen withdrawal (not absence) is related to bone density loss, the supplementation of estrogen in a man should have no effect on his bone mass. From the given answers, only choice (C) correctly mirrors our prediction. (A) Distortion. There is no reason to believe that any change in osteoporosis risk would occur. According to paragraph 1, bone loss is related to estrogen withdrawal, not simply the presence or absence of estrogen. If bone loss was related only to the absence of estrogen, then men would always exhibit a high rate of bone loss. (B) Distortion. There is no reason to believe that any change in osteoporosis risk would occur. According to paragraph 1, bone loss is related to estrogen withdrawal, not simply the presence or absence of estrogen. If bone loss was related only to the absence of estrogen, then men would always exhibit a high rate of bone loss. (D) Distortion. Based on the passage, there is no reason to believe that the disease would occur at an earlier age. 135. (D) Glancing at the answer choices, we notice that our task is to qualitatively describe the changes in osteoblast and osteoclast activity that occur during osteoporosis. We are told in paragraph 1 that osteoporosis is the loss of bone density. In other words, more bone is being destroyed than is being built. Using the definitions given to us in the question stem, we can predict that osteoclast activity (bone resorption) is increased, whereas osteoblast activity (bone formation) is decreased—the correct answer is choice (D). (A) Opposite. Osteoblast activity is decreased, whereas osteoclast activity is increased. (B) Distortion. Osteoblast activity is decreased. (C) Distortion. Osteoclast activity is increased.

67

AAMC Test 3 Explanations

Passage VII (Items 136–141) Passage Map Passage Type: Experiment Topic: Aldol self-condensation of acetone Scope: Characteristics of the product of the aldol self-condensation of acetone ¶1: Starting material + NaOH in acetone resulted in starting material + Product A ¶2: Product A resulted from the aldol self-condensation of acetone ¶3: Characteristics of Product A ¶4: Dehydration of Product A with sulfuric acid results in Product B. 136. (B) The item asks us to calculate the molecular weight of the ketone that undergoes an aldol self-condensation to result in a β-hydroxy ketone with a molecular weight of 144 g/mol. Let’s reason through the reaction mechanism to determine the difference between the reactants and the products of the aldol condensation. When a ketone undergoes an aldol self-condensation reaction, two ketone molecules combine without losing water. In fact, this reaction proceeds without any loss of molecular weight from either ketone reactant. The resulting β-hydroxy ketone is a dimer of the starting molecule as seen in the reaction mechanism below. O–

O

O

OH–

+

O

O O–

H+

OH

Based on this, we can predict that the molecular weight of the final product is exactly double that of the initial reactant. If the molecular weight of the final product was 144 g/mol, then each initial reactant weighed 72 g/mol— choice (B) is the correct answer. Also, we could have reached this conclusion by using the information given to us in the passage. We are told that Product A is the product of a self-condensation reaction, and that it has a molecular weight of 116 g/mol. The passage also mentions that the starting compound is acetone. Acetone’s molecular weight is 58 g/mol (12 × 3 + 6 + 16 = 58). As such, Product A has the molecular weight of two acetone molecules—the product of a self-condensation reaction has a molecular weight that is simply the summation of the molecular weight of the reactants. Either method of analysis shows choice (B) to be the correct answer.

68

Biological Sciences

(A) Miscalculation. This would be the molecular weight of a reactant resulting in a β-hydroxy ketone with a molecular weight of 140 g/mol. (C) Miscalculation. This would be the molecular weight of a reactant resulting in a β-hydroxy ketone with a molecular weight of 148 g/mol. (D) Miscalculation. This would be the molecular weight of a reactant resulting in a β-hydroxy ketone with a molecular weight of 152 g/mol. 137. (C) The item asks us for a way to shift the equilibrium of the aldol self-condensation to favor the formation of products. The concept tested in this question is Le Châtelier’s principle, so let’s quickly review what it implies about chemical reactions. Le Châtelier’s principle states that when a stressor is applied to a reaction (change in concentration, pressure, temperature), the equilibrium of the reaction will shift so as to relieve that stressor. With this in mind, we predict that, in order to shift the reaction equilibrium towards the formation of products, we could either remove the product as it is formed or increase the amount of reactants present. Choice (C), removing Product A as it is formed, will shift the equilibrium toward Product A, and is thus the correct answer. (A) Distortion. Catalysts do not affect the equilibrium, only the rate of reaction. (B) Distortion. Catalysts do not affect the equilibrium, only the rate of reaction. (D) Opposite. Increasing the reaction temperature to the boiling point of acetone will remove the reactant through evaporation, thus shifting the equilibrium to favor the formation of reactants, not products. 138. (D) The item asks us to use only Observations 1 and 2 from the passage to determine a possible structure for Product A. Let’s start by analyzing the two observations. They state that Product A has a molecular weight of 116 g/mol, and is composed only of carbon, hydrogen, and oxygen. Essentially, the correct answer is that which proves to be a constitutional isomer of Product A, which is shown in the passage. Analyzing Figure 1, we see that Product A has 6 carbon atoms, two oxygen atoms, and 12 hydrogen atoms—the only structure that matches this is the molecule found in choice (D), the correct answer. (A) Distortion. This molecule doesn’t contain any oxygen atoms—Observation 1 indicates that the composition of Product A includes oxygen. (B) Distortion. This molecule has a molecular weight of 130 g/mol. (C) Distortion. This molecule has a molecular weight of 100 g/mol. 139. (B) Glancing at the answer choices, the question is asking us to determine if Br2 will react with Product B. What do we know about the reactions with Br2 in CCl4? Observation 5 from the passage tells us that when Br2 is exposed to a molecule that doesn’t contain double bonds, the red bromine color persists. In reality, this question asks us to ascertain whether Product B contains a double bond. What do we know about Product B? In paragraph 3, we are told that Product B is the result of a dehydration reaction of Product A—dehydration products have double bonds. As such, when exposed to Br2 in CCl4, the red color should not persist—Br2 will add across the double bond of Product B. Of the given answer choices, choice (B) indicates that the color will not be red, because Product B contains a double bond, and is the correct answer.

69

AAMC Test 3 Explanations

O

Product B (A) Distortion. The solution becomes colorless because Product B contains a double bond. (C) Opposite. The solution becomes colorless because Product B contains a double bond. (D) Opposite. The solution becomes colorless because Product B contains a double bond. 140. (A) The item is essentially asking us to determine how many proton NMR signals are produced by acetone. Let’s start by quickly reviewing how NMR reflects the structure of a compound. NMR detects the absorption of magnetic energy by a nucleus. In proton NMR, the absorption of energy by a hydrogen nucleus is detected. The absorption of energy is determined by each proton’s chemical environment. When asked how many signals are produced by acetone, we can find the answer by determining how many chemically unique protons are in acetone. Acetone, whose structure is shown below, has 6 total protons, but all are methyl protons located one carbon bond away from a carbonyl atom. Therefore, they are all chemically equivalent, so each proton absorbs the same amount of energy. As a result, acetone produces only one signal in the proton NMR spectrum—the correct answer is choice (A).

H H

O

H H

H H

(B) Distortion. This is the number of signals acetone would produce in

13C

NMR.

(C) Distortion. Since all six hydrogens in acetone are equivalent, they will only produce one signal. (D) Distortion. This is the number of protons found in acetone, or the area under the curve of the one signal produced. 141. (D) We are asked to determine which compound would result in a positive iodoform test. The answer choices refer to Products A and B, and acetone. To begin with, we must understand what an iodoform test tells us about a compound. According to Observation 4, Product A gives a positive iodoform test because it is a methyl ketone. We know from their chemical structures (shown below) that Product B and acetone are also methyl ketones—we would expect all three compounds to result in a positive iodoform test, making choice (D) the correct answer. O O

Product B

70

Acetone

Biological Sciences

(A) Distortion. Product B and acetone will also result in a positive iodoform test. (B) Distortion. Acetone will also result in a positive iodoform test. (C) Distortion. Product B will also result in a positive iodoform test.

Discrete Set III (Items 142–146) 142. (D) We are told that eukaryotes localize the process of oxidative phosphorylation in the mitochondrion—we are asked to find the analogous structure in which this occurs in prokaryotes. In eukaryotic cells, oxidative phosphorylation occurs in the inner membrane of the mitochondrion. The generally accepted scientific theory is that mitochondria were once bacteria that had become engulfed by eukaryotic cells, forming a symbiotic relationship. As such, the inner mitochondrial membrane is analogous to a bacterial plasma membrane. Indeed, the bacterial plasma membrane contains all of the necessary proteins for oxidative phosphorylation. (A) Distortion. The cell wall maintains the shape of the cell. (B) Distortion. The ribosome synthesizes proteins from mRNA. (C) Distortion. Bacterial cells do not contain a nuclear membrane, as bacteria do not have membrane-bound organelles. 143. (A) This is a straightforward item that asks us to remember where uracil is incorporated into nucleic acids. Knowing that all the nucleic acids in eukaryotic cells are synthesized in the nucleus allows us to select choice (A) as the correct response. Uracil is a pyrimidine analogous to thymine. Although thymine is a base that is part of DNA, uracil is only present in RNA. The correct answer, again, is choice (A). (B) Distortion. The Golgi body packages and distributes proteins and other newly synthesized molecules. (C) Distortion. Ribosomes are involved in the synthesis of proteins. (D) Distortion. The endoplasmic reticulum is the site of synthesis of secreted proteins and lipids. 144. (D) The question asks us to explain why the activity of pepsin decreases below a pH of 1.0. Pepsin is an enzyme, and like all other enzymes, it has an optimal temperature and pH at which it functions best. These are the conditions under which the enzyme structure is most favorable for catalyzing the designated reaction. Outside of these conditions, the enzyme denatures, or changes its three-dimensional conformation. As a result, the active site of the enzyme is altered and the enzyme cannot function. Our line of reasoning matches best with choice (D), the correct response. (A) Distortion. The acidic environment affects the structure of the protein—it does not act as an inhibitor, rather, it denatures the protein. (B) Distortion. Pepsin synthesis is stimulated by acid production. (C) Opposite. Peptide bonds may break under these acidic conditions.

71

AAMC Test 3 Explanations

145. (C) This item asks us to determine the maximum number of possible combinations in which the three amino acids in the figure can be arranged. To determine the number of configurations possible, we can use a factorial (n!) calculation. There are 3 amino acids, so the number of possible conformations is 3! = 3 × 2 × 1 = 6. The correct answer is therefore choice (C). A lengthier method to calculate the number of tripeptides possible is to simply denote each of the three amino acids with a letter, A, B, and C, writing all the different combinations. Using these designations, the possible conformations are ABC, ACB, BAC, BCA, CAB, CBA, for a total of 6 different conformations. Again, choice (C) is the correct answer. (A) Miscalculation. There is more than one tripeptide combination possible. (B) Miscalculation. This is the number of peptides in the molecule. (D) Miscalculation. This answer would be incorrectly obtained if you squared the number of peptides. 146. (B) The item asks us to determine what the effect of inhibiting spindle microtubules would be on the daughter cells of a plant. What is the role of tubulin molecules? Spindle microtubules function in mitosis to align and separate the chromosomes. Inhibiting them would result in unequal division of chromosomes between daughter cells. If this unequal division continued, some of the resulting plant cells would have more than two sets of chromosomes. The answer choice that matches with our prediction is choice (B). (A) Distortion. Genetic variability is created by crossing over in mitosis, not the spindle apparatus. (C) Distortion. Microtubules and tubulin molecules are not involved in the cell wall. (D) Distortion. The assembly and disassembly of microtubules results in movement; this drug inhibits microtubule assembly, therefore preventing movement.

72